You are on page 1of 78

ACTIVE SITE EDUTECH - 9844532971

Sr |12th class CHEMISTRY VOL-2

CHEMICAL KINETICS
Chapter
4

INTRODUCTION within the fraction of seconds are called very


fast reactions.
 Chemical Kinetics is the branch of chemistry Ex:-1) Neutralization between strong acids
that deals with and strong bases.
a) Rate of reactions
b) Factors influencing the rate of reaction NaOH ( aq )  HCl( aq )  NaCl( aq )  H 2O( l )
c) Reaction mechanism * 2) Precipitation reactions
* The word kinetics is derived from the Greek NaCl( aq )  AgNO3( aq )  AgCl( s )   NaNO3( aq )
word ‘kinesis’, which means movement.
3) Explosive reactions :
* Thermodynamics tells about only the feasibility
Explosion of T.N.T (Tri nitro toluene)
of a chemical reaction where as chemical
b) Moderate reactions: The chemical
kinetics tells about the velocity of reaction.
reactions which are completed in
* Example: Thermodynamic data indicates that
measurable time are called as moderate
diamond shall be converted to graphite but in
reactions.
reality the conversion rate is so slow that the
* Ex:- 1) Inversion of cane sugar
change is not perceptible at all.
* Kinetics studies not only help us to determine C12 H 22O11( aq )  H 2O( l )  C6 H12O6 ( aq )  C6 H12O6 ( aq )
glucose fructose
the speed (or) rate of a chemical reaction but
also describe the conditions by which the 2)Combustion of hydrogen (or) coal [under
reaction rates can be altered. normal conditions].
 Based on the velocity of chemical reactions, 1
H 2 ( g )  O2 ( g )  H 2O( g )
the reactions are classified into three types. 2
a) Very fast (or) instantaneous reactions: 3) Decomposition of hydrogen peroxide
The chemical reactions which are completed
2H 2 O 2 (l ) 
 2H 2 O(l )  O 2 (g )

CHEMICAL KINETICS 1
ACTIVE SITE EDUTECH - 9844532971
ACTIVE SITE EDUTECH - 9844532971

Sr |12th class CHEMISTRY VOL-2


c) Very Slow Reactions : The chemical
reactions which complete in very long time = (Average rate) lim
t 0

are called very slow reactions.


* Ex -1 :-Rusting of Iron in presence of air and  [ A]   [ B ] 
rinst    lim   
 t  t 0  t lim or
moisture t  0

4 Fe  3O2  xH 2O  2 Fe2O3 .xH 2O. d [ A] d [ B]


Ex -2 :- Fermentation of sugar in presence of rinst  ( )  ()
dt dt
yeast takes more than a month. Hence, Slope of the tangent at time ‘t’ in plot
 It is not possible to determine the rates of very of concentration with time gives instantaneous
fast and very slow reactions by conventional rate of reaction.
methods.But the rates of reactions with Instantaneous rate of reaction =
moderate speed can be determined.
 C  dC
Rate of reaction: lim   
 t 
t  0 dt
; rinst  lim
t 0
 raverage 
* Rate of reaction defined as the change in
concentration of reactant or product per unit
time. it is always a positive quantity.Where
dC = change in concentration in a small interval
dt [+] sign is used when we refer for product
concentration.
[-] sign is used when we refer for reactant
concentration.
* Units Of Rate Of Reaction:
mol.lit-1.time-1
(mol L-1 s-1 or mol L-1 min-1 or mol L-1 h-1)
* Average Rate of Reaction:
The rate of reaction over a certain measurable
period of time during the course of reaction is
called average rate of reaction. it is denoted
by ‘r’.
For a reaction A B

 [A]2  [A]1  [A]


raverage    
 t 2  t1  t
Where [A]1 = Concentration of reactant ‘A’ at
time ‘t1’
[A]2 = Concentration of reactant ‘A’ at time
‘t2’
* Instantaneous Rate of Reaction:
The rate of reaction at any particular instant RATE OF REACTION IN FORM OF
during the course of reaction is called STOICHIOMETRY OF A CHEMICAL
instantaneous rate of reaction. REACTION:
For a reaction A B Let us consider a reaction :
Mathematically: Instantaneous rate m1 A  m2 B  n1P  n2Q
Where,

2 CHEMICAL KINETICS
ACTIVE SITE EDUTECH - 9844532971
ACTIVE SITE EDUTECH - 9844532971

Sr |12th class CHEMISTRY VOL-2

d [ A] given below, calculate the average rate of


Rate of disappearance of A   the reaction:
dt
d [ B] C4 H 9Cl  H 2O  C4 H 9OH  HCl during
Rate of disappearance of B  
dt the different intervals
d[P] t/s 0 50 100
Rate of appearance of P 
dt
C4 H 9Cl  mol L1 0.100 0.0905 0.0820
d [Q ]
Rate of appearance of Q  150 200 300 400
dt
Rate of reaction
0.0741 0.0671 0.0549 0.0439
1  d [ A]  1  d [ B]  1 d [ P] 1 d [Q] 500 700 800
     
m1  dt  m2  dt  n1 dt n2 dt 0.0335 0.0210 0.017
Rate of reaction is always positive, negative sign Sol: We can determine the difference in
represents decrease in concentration of concentration over different intervals of time and
reactant. thus determine the average rate by dividing
EXAMPLE-1: The decomposition of N 2O5 in
  R  by t
CCl4 at 318K has been studied by Average rates of hydrolysis of butyl
monitoring the concentration of N O in chloride
2 5

the solution. Initially the concentration of 4 –1 –1


rav × 10 /mol L s
N 2O5 is 2.33 molL and after 184 minutes,
1
C 4 H 9Cl t1 / C4 H9Cl t2 / t1/s t2/s = {[C4H9Cl]t2 – [C4H9Cl]t1
it is reduced to 2.08 molL1 . The reaction mol L1 mol L1 / (t2 – t1)} × 104
takes place according to the equation 0.100 0.0905 0 50 1.90
2 N 2O5  g   4 NO2  g   O2  g  . 0.0905 0.0820 50 100 1.70
Calculate the average rate of this reaction 0.0820 0.0741 100 150 1.58
in terms of minutes? What is the rate of
0.0741 0.0671 150 200 1.40
production of NO2 during the period?
Sol: Average rate 0.0671 0.0549 200 300 1.22

1    N 2O5   1   2.08  2.33 molL  0.0549 0.0439 300 400 1.10


1

       
2 t  2 184 min  0.0439 0.0335 400 500 1.04
 6.79  104 molL1 / min It may be understand 0.0210 0.017 700 800 0.4
that
It can be seen that the average rate falls from
1   NO2  1.90 104 mol L1s 1 to 0.4 104 mol L1s 1
Rate   6.79  104 molL1 / min
4 t
Instantaneous rate: The average rate can not
  NO2  be used to predict the rate of reaction at
 4  6.79  104 molL1 min 1
t particular instant as it would be constant for
 2.72 103 molL1 min 1 the time interval for which it is calculated. So,
EXAMPLE-2: From the concentrations of to express the rate at a particular moment of
C4 H 9Cl (Butylchloride) at different times time we determine the instantaneous rate.

CHEMICAL KINETICS 3
ACTIVE SITE EDUTECH - 9844532971
ACTIVE SITE EDUTECH - 9844532971

Sr |12th class CHEMISTRY VOL-2


Instantaneous rate of hydrolysis of What is the average rate of decomposition
butylchloride of A2 B ? Assume T and V are constant.
Sol: The decomposition reaction of gaseous A2 B
1
is given as A2 B  A2  B2
2
100 0 0 initial reaction
100-2x 2x x final reaction
100 -2x + 2x + x = 100 + x =120mm
x = 20 mm or 2x = 40 mm
The decrease in pressure of reactant substance
A2 B in 5min is 40mm.
40
It can be determined graphically by drawing a The rate of decomposition of A2 B 
5
tangent at time ‘t’ on either of the curves for
concentration of R and P vs time ‘t’ and  8mm min 1  0.133mms 1
calculating its slope. rinst 600s , can be EXAMPLE-5
calculated by plotting concentration of butyl A chemical reaction 2A   4B  C in gas
chloride as a function of time. A tangent is drawn phase occurs in a closed vessel. The
that touches the curve at t  600 s (from the concentration of B is found to increase by
above graph) 5 × 10–3 mol L–1 in 10 seconds. Calculate
The slope of this tangent gives the instantaneous (i) the rate of appearance of B (ii) the rate
of disappearance of A.
rate. so rinst at 600s (1) 5×10–4 mol–1L–1sec–1
 0.0165  0.037  1 5 1 1
(2) 1.25×10–4mol–1L–1sec–1
    mol L  5.12  10 mol L s (3) 2.5×10–4 mol–1L–1sec–1
  800  400  s 
(4) 6.25×10–4mol–1L–1sec–1
EXAMPLE-3: N 2  3H 2  2 NH 3 the rate of Sol:
2A   4B  C (Given reaction)
disappearance of nitrogen is
1 d[A] 1 d[B] d[C]
0.02molL1s 1 . What is the rate of Rate =   
2 dt 4 dt dt
appearance of ammonia? Rate of appearance of
d  N 2  1 d  NH 3  1 d  NH 3  (5  10 3 mol L1 )
Sol:  ; =0.02 B  5  10  4 mol L1s 1
dt 2 dt 2 dt (10 s)
d  NH 3  Rate of disappearance of
 0.04 mol.L1s 1  d[A] 2 d[B] 1 d[B]
dt A  
The rate of appearance of ammonia dt 4 dt 2 dt
= 0.04molL1s1 . 1
=  (5  10  4 mol L1s 1 ) = 2.5 × 10–4 mol L–1
2
EXAMPLE-4: A2 B is an ideal gas, which sec–1.
decomposes according to the equation Factors Affecting The Rate Of
1 Reaction
A2 B  A2  B2 . At start, the initial a) Nature of the reactants:
2
Reactions between ionic substances take place
pressure is 100mm of Hg and after 5 much faster than the reactions occuring between
minutes, the pressure is 120mm of Hg . covalent substances.Because in ionic reactions

4 CHEMICAL KINETICS
ACTIVE SITE EDUTECH - 9844532971
ACTIVE SITE EDUTECH - 9844532971

Sr |12th class CHEMISTRY VOL-2


there is no breaking and making of bonds. greater will be the possibility of their collisions
NaCl aq   AgNO3 aq   AgCl s    NaNO3 aq  and higher will be the rate of reaction.
 The reactions between covalent molecules Eg:- When zinc pieces are added to dilute
involve the breaking (cleavage) and the making HCl , chemical reaction takes place slowly
(formation) of covalent bonds.
liberating H gas. But the same reaction is
H 2
Eg : C2 H5OH l   CH3COOH l  

rapid by taking concentrated HCl .
CH3COOC2 H5 l   H 2O l   From the given below graph it is clear that, the
 Reactions which involve lesser bond rate of reaction gradually decreases with time
rearrangements are rapid at room temperature because of the decrease in the concentration
than those which involve more bond of reacting substances with time.
rearrangements.
a) 2 NO  O2  2 NO2
b) CH 4  2O2  CO2  2 H 2O
 Reaction (a) involves breaking of 6 bonds and
formation of six bonds
O
||
2  N  O   O  O  2 N O
(4) (2) (6)
c) Effect of temperature on the reaction Rate :
 Reaction (b) involves breakage of 8 bonds and
formation of eight bonds. The rate of a reaction increases with increase
in the temperature.
H H  In most cases a rise of 100C in temperature
| | generally doubles the specific rate of the
H  C  H  2 O  O  O  C  O  2O H reaction.
| * Increasing the temperature of the substance
H increases the fraction of molecues, which collide
(4) (4) (4) (4)
with energies greater than activation energy
 Reaction (a) is faster than reaction (b)
b) Concentration of the reactants:
 Ea  . Hence the rate of reaction increases.
Except zero order reactions, for all other
reactions the rate depends on the concentration
of the reactants.
 rate  (concentration of the reactants)n
 dC   dC 
(or)     C (or)     kC
n n

 dt   dt 
‘n’ may be any simple value including zero.
 For gaseous reactants. rate  (pressure of the
reactants)n
dP  dP
(or)   P n or  kP n
dt dt
 Chemical reactions occur due to the collisions
among the reacting molecules. Hence greater
the number of these molecules in unit volume,

CHEMICAL KINETICS 5
ACTIVE SITE EDUTECH - 9844532971
ACTIVE SITE EDUTECH - 9844532971

Sr |12th class CHEMISTRY VOL-2


 The ratio of two specific rates measured at
k2 Ea  1 1 
temperature that differ by 10o C is called the ln    
k1 R  T1 T2 
Temperature co-efficient
kt oC 10 k2 Ea  1 1 
2 log    
 Temperature co-efficient = k k1 2.303R  T1 T2 
t oC

(Normally t o C  25o C; t o C  10  35o C ) Ea 1


 log k  log A 
2.303R T
For some reactions the ratio approaches the
value of 3 Ea
Slope = 
 Hydrogen and oxygen combine very slowly at 2.303R
ordinary temperature but rapidly at high (R=gas constant)
temperature.
 A piece of coal will burn rapidly in air when the
temperature is raised sufficiently.
* Arrhenius equation for temperature
dependence of a rate constant.
 Ea
k  A e RT
k = Rate constant,
A = Arrhenius frequency constant d. Effect of Catalyst :
Ea = Activation energy,  i) Presence of positive catalyst: The function
R= Gas constant of a positive catalyst is to lower down the
T = absolute temperature activation energy.
 Ea The greater the decrease in the activation
e RT = fraction of moledules with K.E > Ea energy higher will be the rate of reaction
 Ea In the presence of a catalyst, the reaction
k  A.e RT
follows a path of lower activation energy.
Taking ‘ln’ on both sides
In this condition, a large number of reacting
 Ea
ln k   ln A  1 molecules are able to cross over the energy
RT barrier and thus the rate of reaction increases.
at temperature T1 equation 1 is

 Ea
ln k1   ln A   2
RT1

at temperature T2 equation 1 is

 Ea
ln k2   ln A   3
RT2
( since A is constant for a given reaction)
From equations (2) and (3)
Ea E
ln k2  ln k1   a
RT1 RT2

6 CHEMICAL KINETICS
ACTIVE SITE EDUTECH - 9844532971
ACTIVE SITE EDUTECH - 9844532971

Sr |12th class CHEMISTRY VOL-2


ii) Presence of negative catalyst: A negative
k2 Ea  T2  T1 
catalyst increases the activation energy of log   
reaction by forming a new intermediate of high k1 2.303R  T1T2 
energy, i.e., by changing the reaction Substituting the values k1 , k2 , T1 , T2 and molar
mechanism. Due to increase in activation
gas constant R
energy, some active molecules become
inactive, therefore, rate of reaction decreases. Ea  1000 10
0.301  
2.303  8.314 300  310
Let k p denote presence of catalyst and ‘ ka ’
Energy of activation  Ea  53kJmol 1
denote absence of catalyst.
 E p / RT
EXAMPLE-7: The temperature coefficient of
k p  Ae ; k a  Ae  Ea / RT a reaction is 2 and the rate of reaction at
deviding eq.(1) by eq. (2) we get 250 C is 3molL-1min -1 . Calculate the rate
kp  Ea  E p  / RT at 750 C
e e E / RT ;
ka Sol: According the Arrhenius, for every 100 C rise
in the temperature, the rate of reaction is
E
kp  E  doubled.
 anti log    10 2.303 RT
ka  2.303 RT  Rise in temperature = 750 C  250 C  500 C
 Catalyst alters i) rate of reaction
ii) path of reaction iii) activation energy  5  100 C
iv) threshold energy v) rate constant The increase in the rate of the reaction
 Catalyst does not alter  25  32
i)  G of reaction The rate of reaction
ii) energy of reactants and energy of products 750 C  3  32  96mol L1 min 1
iii)  H iv) S v) kC Characteristics Of Catalyst
 In some reactions, the rate of the reaction is i) A catalyst remains unchanged chemically at
directly proportional to the concentration of the the end of reaction, however its physical state
catalyst. may change. e.g. MnO2 used as catalyst in
Ex: Acid catalysed hydrolysis reactions of granular form for the decomposition of KClO3,
esters, the rate is proportional to the is left in powder form at the end of reaction.
concentration of the acid catalyst. ii) A catalyst never initiate a chemical reaction.
* Catalyst increases the rate of reaction by It simply influences the rate of reaction.
making an alternate path of low activation Exception: combination of H2 and Cl2 takes
energy for reactant molecules. place only when moisture (catalyst) is present.
Ex:- 2 KClO3  MnO2
 2 KCl  3O2 iii) A small quantity of catalyst is sufficient to
 Note: Pressure of the gases, volume of the influence the rate of reaction e.g. 1 g atom of
vessel, state of the substance, pH etc also Platinum is sufficient to catalyse 108 litre of
influence the rate of some reactions. H2O2 decomposition.
EXAMPLE-6: At 27 0 C and 37 0 C , the rates iv) A catalyst does not influence the equilibrium
of a reaction are given as constant of reaction. It simply helps in attaining
1.6  102 molL1s 1 and equilibrium earlier. It alters the rate of forward
and backward reactions equally.
3.2  102 molL1s 1 . Calculate the energy
v) Catalyst’s activity is more or less specific. A
of activation for the given reaction.
Sol: The ratio of specific rates at two different catalyst for one reaction is not necessary to
temperatures are given as, catalyse the another reaction.
CHEMICAL KINETICS 7
ACTIVE SITE EDUTECH - 9844532971
ACTIVE SITE EDUTECH - 9844532971

Sr |12th class CHEMISTRY VOL-2


Rate Law :  NOCl  NO2  Cl (slow step)
2
The mathematical expression of the relation
between rate of reaction and concentration of  NO2Cl  Cl  NO2  Cl2 (fast step)
reactants is known as rate law. So the rate law from slowest step,
In this expression the rate of a reaction is Rate = k[ NO2Cl ]
proportional to the product of molar In this way the predicted rate law derived from
concentration of reactants with each term raised two step mechanism agrees with experimental
to a power or exponent that has to be found rate law.
experminetaly. EXAMPLE-9:
In a chemical reaction: aA  bB  Product
2 NO( g )  2 H 2( g )  N 2( g )  2 H 2O( g )
The rate law is: Rate  [ A]x [ B ] y
experimentally the rate law is, Rate =
The values of exponents x and y are found
experimentaly which may or may not be same k[ NO]2 [ H 2 ]
as stoichiometric coefficients. The mechanism of the reaction is given as
Above relationship can be written as:  2 NO  N 2O2 (fast step)
Rate  k[ A] [ B]
x y
 N 2O2  H 2  N 2O  H 2O (slow step)
Where k is a proportionality constant known
 N 2O  H 2  N 2  H 2O (fast step)
as rate constant or specific rate.
MECHANISM OF REACTION: The Rate law from slowest step is:
 Elementary reactions: Rate = k[ N 2O2 ][ H 2 ]
Those reactions which completes in single step The rate law expression should be free from
and which have exponents in rate law equal to species N2O2.
stoichiometric coefficients of the reactants. From fast reversible step -
 If A  B  Products ; is an elementary
k f [ NO]2  kb [ N 2O2 ]
reaction.
then rate law will be k
Rate = k[A][B] [ N 2O2 ]  f [ NO]2
kb
 Zero order reactions can never be elementary
reactions. and rate law becomes Rate =
 For elementary reactions fractional order is not
 kf 
possible. k   [ NO]2 [ H 2 ]
 Complex reactions:  kb 
These reactions will complete in multisteps. For therefore, Rate = k '[ NO]2 [ H 2 ]
these reactions a mechanism is proposed.
 For complex reactions the overall rate of This derived rate law agrees with experimental
reaction is controlled by the slowest step which rate law.
is called rate determining step. EXAMPLE-10:
 In rate law expression rate of reaction depends 1)Nitrogen dioxide reacts with hydrogen
on concentration of reactants of slowest step to give nitrogen and water according to the
which must be free from intermediate. equation :
EXAMPLE-8: 2 NO Cl  2 NO  Cl 2 NO 2  H 2   N 2  H 2 O 2 (slow)
2 2 2
Expermintally, the rate law is Rate =  2H 2 O (fast)
H 2  H 2O 2 
k[ NO2Cl ] The mechanism of the reaction What is the predicted rate law?
is given as (1) Rate = k[NO2] (2) Rate = k[H2]2

8 CHEMICAL KINETICS
ACTIVE SITE EDUTECH - 9844532971
ACTIVE SITE EDUTECH - 9844532971

Sr |12th class CHEMISTRY VOL-2


(3) Rate = k[NO2]2[H2] 1
iv)H 2O2 aq   H 2O l   O2 g  ; r  k  H 2O2 
(4) Rate = k[NO2][H2] 2
Sol: The rate law or rate equation is given by the v) CH3COOC2 H5 aq  NaOH aq 
first step which is the slow step. Therefore,
CH3COONa aq  C2 H5OH aq
predicted rate law for the reaction is
Rate = k [NO2]2 [H2] r = k CH 3COOC2 H 5 
1
 NaOH 
1

EXAMPLE-11:
For the reaction * vi) CHCl3  Cl2  CCl4  HCl
NO2(g) + CO(g)   NO(g) + CO2 (g), Rate = k CHCl3 Cl2 
1/ 2

the experimentally determined rate


* vi i)
expression at 400 K is :rate = k [NO2]2
What is the proposed mechanism for the CH 3COOC2 H 5  H 2O  CH 3COOH  C2 H 5OH
reaction?
Rate = k CH 3COOC2 H 5   H 2O 
1 0

Sol: According to available information, rate is


proportional to the square of the molar con Characteristics of Rate constant:
centration of NO2. Therefore, the proposed (i) Rate constant is a measure of the rate of
mechanism for the reaction is : the reaction. Greater the value of k, faster
NO 2  NO 2 (
Slow )
 NO  NO 3 is the reaction. Similarly, smaller the value
of k indicates slower the reaction.
NO 3  CO (
Fast )
 CO 2  NO 2
(ii) The value of k depends on the nature of
Rate Constant: The rate constant of reaction the reactants. It is a characteristic constant
becomes equal to the rate of the reaction when for a particular reaction at a fixed
the concentration of all the reactants are unity, temeperature. Different reactions have
hence the rate constant is also known as the different values of k.
specific reaction rate. (iii) The value of the rate constant of a reaction
 A + B  Products. does not depend on the concentrations of
 If the initial concentration of B is taken in large the reactants.
excess than A, then rate depends on A only. (iv) The value of rate constant for a particular
 nA  mB  products reaction changes with temeperature.
(v) The units of rate constant depend on the
Rate r   A n
 B m order of reaction.
rate = k  A n  B  0  Rate law equation for reversible reaction
k1
 'k' is called rate constant or specific rate or rate 
H 2  I 2  
 2HI
k2

per unit concentrations of the reactants.


Rate = k 1  H 2  I 2   k 2  H I 
2

rate
k Rate of forward Rate of backward
 reactants    reaction
n
Rate = reaction 
   

Units of k  mole1 n Ln 1 sec1  Parallel or competing reaction


The reaction in which a substance reacts or
i) 2 NO  Cl2  2 NOCl ; r  k  NO  Cl2 
2
decomposes in more than one way are called
side reactions or parallel reactions.
ii) SO2Cl2  SO2  Cl2 ; r  k  SO2Cl2 

iii) 2 HI  g   H 2 g   I 2 g  ; r  k  HI  and
2

H 2 g   I 2 g   2 HI  g  ; r  k  H 2  I 2 

CHEMICAL KINETICS 9
ACTIVE SITE EDUTECH - 9844532971
ACTIVE SITE EDUTECH - 9844532971

Sr |12th class CHEMISTRY VOL-2

 d  A Molecularity of the reactions:


  k1  k2   A  kav  A
dt  The sequence of the elementary steps of a
chemical reaction is known as reaction
k1 =fractional yield of B  kav
mechanism.
k2 =fractional yield of C  kav  The slowest elementary step of the reaction is
If k1  k2 then, called rate determining step or rate limiting step.

A  B main and A  C is side reaction * The number of atoms (or) molecules (or) ions
Let after a definite interval x mol/litre of B and participating in the slowest step is called
y mol/litre of C are formed. molecularity.

x k1 * Molecularity cannot be zero (or) fraction. It is


 always a whole number (or) integer.
y k2
 Rate law equation involving side reactions. * Molecularity is obtained from reaction
mechanism.
Eg : NH 4 NO 2  N 2  2H 2 O Unimolecular

2HI  H 2  I 2 Bimolecular

Rate of formation of Th 227  k1  Ac 227  2NO  O 2  2NO 2 trimolecular

Rate of formation of Fr 223  k 2  Ac 227   The probability that more than three molecules
can collide and react simultaneously is very
Rate   k1  k 2   Ac 227 
small. Hence, the molecularity greater than three
Order of the reaction : is not observed. It is, therefore, evident that
* The sum of the powers of the concentration complex reactions involving more than three
terms of reactants in the rate equation is called molecules in the stoichiometric equation must
order of the reaction. take place in more than one step.
* Order of reaction may be zero (or) fraction (or)
negative (or) a whole number (or) integer * KClO3  6 FeSO4  3H 2 SO4 
 Order of the reaction can be determined by
KCl  3Fe2  SO4 3  3H 2O
experimental method only.
 For elementary reactions order can be obtained This reaction which apparently seems to be of
from stoichiometric equation. tenth order but it is actually a second order
Note: A Chemical reaction which occur in reaction. This shows that the reaction takes
single step is called elementary reaction. place in several steps. And the rate determining
Ex 1:- xA  yB  zC  products; step involves just two species.
*3 Consider the decomposition of hydrogen
R  k  A   B C 
x y z
order  x  y  z
peroxide which is catalysed by iodide ion in an
alkaline medium.
1 3
* Ex 2:- A  B  products
2 2 2H 2O2  I

Alkalinemedium
 2H 2O  O2
1 3 The rate for this reaction is found to be
; Rate = k  A  B  2 ; Order =
1 3
2  2
2 2

10 CHEMICAL KINETICS
ACTIVE SITE EDUTECH - 9844532971
ACTIVE SITE EDUTECH - 9844532971

Sr |12th class CHEMISTRY VOL-2

d  H 2O2  2 HI 
Au
 H 2  I2
Rate =  k  H 2O2   I  
dt Ex. 3:- Decomposition of N2O on a hot
This reaction is first order with respect to both Platinum surface.
H 2O2 and I  . Evidences suggest that this 1
N2O  N2  O2
reaction takes place in two steps. 2
Difference Between Molecularity and Ex. 4 :- Photochemical combination reaction
order of React: of H2 and Cl2 to form HCl
h
H2  Cl2   2HCl

Rate law for Zero Order Reaction:


A
 Products
The instantaneous reaction rate for zero order
d [A] d[A]
reaction is :   k[A]0 or k
dt dt
–d[A] = kdt (or) d[A] = –kdt
Integrating both sides,

 d[A] 
  k dt or [ A ]   kt  I ...... (i)

Here ‘I’ is the constant of integration. The value


of ‘I’ can be calculated from initial conditions,
when t = 0, [A] = [A]0
 [A]0 = – k × 0 + I or I = [A]0
Substituting the value of ‘I’ in eqn. (i)
[A] = –kt + [A]0 or kt = [A]0 – [A]
ZERO ORDER REACTION
A zero order reaction implies that the reaction (or)  A    A 0  kt
rate is proportional to the zero power of the
It is the rate law equation for zero order reacion
concentration of the reactants.
A zero order reaction implies that the reaction
rate is independent of the concentration of the
reacting species.
These reactions are mostly of heterogeneous
nature carried on the catalyst surface.
* Ex. 1:- The thermal decomposition of ammonia
on the surface of hot Platinum catalyst.
2 NH 3 
Pt
 N 2  3H 2
* Ex. 2:- The thermal decomposition of HI on
gold surface.
CHEMICAL KINETICS 11
ACTIVE SITE EDUTECH - 9844532971
ACTIVE SITE EDUTECH - 9844532971

Sr |12th class CHEMISTRY VOL-2

CH3COOC2H5  H2O 
CH3COOH  C2H5 OH
(pseudo first order reaction)
Ex. 4:- Decomposition of H2O2 :
2H2O2  2H2O  O2
Ex. 5:- Decompostion of NH4OH:
NH4OH  N2O  2H2O
Ex. 6:- Radioactive disintegration:
87
36 Kr 86 1
36 Kr  0 n

Half Life for Zero Order Reaction * Ex. 7:- Decomposition of N2O5
* For a zero order reaction, the half life period is 2 N 2O5  g   2 N 2O4  g   O2  g 
directly proportional to the initial concentration
* Ex. 8:- Decomposition N2O
of reactant and is inversely proportional to rate
constant. 2 N 2O  g   2 N 2  g   O2  g 
[A]0  [A] [A]0  [A] Integrated expression
k or t 
t k Let us consider the reaction
[A]0  1 / 2[A]0 [A]0 A
 Products
t1/ 2   If reaction follows first order kinetics, then the
k 2k
instantaneous reaction rate may be expressed
Thus, t 1/ 2  [ A ]0 and t 1/ 2  1 / k
as :
 A 0 d[A]
i.e. t1/2    k [A] (k is the rate constant)
2K dt
Rearranging the equation, we get
d[ A ]
  k dt
[A]
On integrating the above equation
d[ A ]
  [A]
k  dt or  ln [A]  kt  I ......(i)

where ‘I’ is the constant of integration. The


FIRST ORDER REACTION value of ‘I’ can be calculated from initial
In a first order reaction, the rate of reaction is conditions.
directly proportional to the concentration of the When t = 0, [A] = [A]0 i.e. initial concentration
one reactant. of A. Thus eqn.(i) becomes,
* Ex. 1:- Hydrogenation of ethene – ln [A]0 = I ......(ii)
C2 H 4  g   H 2  g   C2 H 6  g  Substituting the value of ‘I’ in eqn. (i), we get
– ln [A] = kt – ln [A]0 ln [A]0 – ln [A] = kt
Rate = k C2 H 4 
[ A ]0 1 [ A ]0
Ex. 2:- Inversion of cane sugar : or ln [ A ] = kt or k = ln [ A ] ......(iii)
t
C12H22O11  H2O  C6H12O6  C6H12O6
Glucose Fructose
2.303  A 0
or k  log (  ln = 2.303
Rate = k C12 H 22O11  t [A]
(pseudo first order reaction) log10)......(iv)
Ex. 3:- Acid hydrolysis of ester :

12 CHEMICAL KINETICS
ACTIVE SITE EDUTECH - 9844532971
ACTIVE SITE EDUTECH - 9844532971

Sr |12th class CHEMISTRY VOL-2

2.303 a
or k  t
log
ax
where [A]0=‘a’ mole
L–1
[A] = (a – x) mole L–1
x is the mole L–1 of reactants get changed into
products.
This is called integrated rate equation for the
first order reaction.
 A  kt Interval Formula:
Exponential Form: A  e or  A   A0 e kt
 0 2.303 a  x1
k log
Different Graphs for First Order Reaction t2  t1 a  x2
Where x1 and x2 be the amounts consumed up
to the time t1 and t2 respectively.
Time required for the completion of definite
fraction of the first order reaction is independent
of the initial concentration of the reactant.
For First Order Growth Kinetics:
It it used in population growth and bacteria
multiplication
2.303 ax
k log
t a
When ‘a’ is initial population and  a  x  is
population after time ‘t’
 Pressure change method: This method is
used for gaseous first order reaction. The
calculation of rate constant or time or other
required terms depends on the stoichiometry
of the reaction and the condition for given
pressure. To analyse this, some cases are
discussed for a random gaseous first order
reaction at constant volume and temperature
condition which is given as:
A (g)   B (g) + C(g)
Case - I:
Let for the given reaction initial pressure of A is
P0 and pressure of A after time t is Pt
A(g)   B(g) + C(g)
Initial pressure at t = 0 P0 0 0
Pressure at time (t) (P0 – x) x x
(Here x is number of moles of A which change
to product)
Initial pressure of A = P0

CHEMICAL KINETICS 13
ACTIVE SITE EDUTECH - 9844532971
ACTIVE SITE EDUTECH - 9844532971

Sr |12th class CHEMISTRY VOL-2

Pressure of A at time (t) = Pt = (P0 - x) EXAMPLE-12:- The initial concentration of


2.303 P0 N2O5 in the following first order reaction
Thus, k = log
t Pt N 2O5 ( g )  2 NO2 ( g )  1/ 2O2 ( g ) was
Case - II : 1.24  102 mol L1 at 318 K. The
Let for the given reaction initial pressure of A is
P0 and total pressure after time t is Pt concentration of N 2O5 after 60 minutes
A(g)   B(g) + C(g) was 0.20  102 mol L1 . Calculate the rate
Initial pressure at t = 0 P0 0 0 constant of the reaction at 318K.
Pressure at time (t) (P0 – x) x x Sol: For a first order reaction
Initial pressrue of A = P0 [ R ]1 k (t2  t1 )
Total pressure (Pt) at time (t) = P0 – x + x + x log 
[ R ]2 2.303
= P0 + x
 x = Pt – P0 2.303 [ R ]1
 P0 – x = P0 – (Pt – P0) = 2P0 – Pt k log
(t2  t1 ) [ R ]2
Thus, k  2.303 log P0
t (2P0  Pt ) 2.303 1.24 102 mol L1
 log
60 min  0 min 0.20 102 mol L1
Case - III :
Let for the given reaction total pressure at time 2.303
t is Pt and total pressure after long time is P’  log 6.2 min 1
60
A(g)   B(g) + C(g)
k  0.0304 min 1
Initial pressure at t = 0 P0 0 0
EXAMPLE-13: The following data were
Pressure at time (t) (P0 – x) x x obtained during the first order thermal
Pressure after long time 0 P0 P0 decomposition of N2O5 (g) at constant
Initial pressrue of A = P0 volume:
Total pressure(Pt) at time (t) = P0– x + x + x = P0 + x
2N 2 O5 (g)  2N 2 O 4 (g) + O 2 (g)
 x = Pt – P0
 Pressure of A at time t: P0 – x = P0 –
S.No. Time/s Total Pressure/
(Pt – P0) = 2P0 – Pt (atm)
Pressure after long time P’ = 2P0 1. 0 0.5
2. 100 0.512
P0 = P’/2
Calculate the rate constant.
Thus,
Sol: Let the pressure of N2O5(g) decrease by 2x
2.303 P'/2 P'/2 P' atm. As two moles of N2O5 decompose to give
k= log 0
 =
t (2P - Pt ) P' - Pt 2(P' - Pt ) two moles of N2O4(g) and one mole of O2(g),
Negative Order Reactions: the pressure of N2O4(g) increases by 2x atm
Conversion of ozone into oxygen. and that of O2 (g) increases by x atm.
2N2O5(g)  2N2O4(g) + O2(g)
2 O3  3 O2
Start t=0 0.5 atm 0 atm 0 atm
 O2  O ( fast )
Step1: O3  At time t (0.5 - 2x) atm 2x atm x atm
Pt = PN 2O5 + PN 2O4 + PO2
Step2: O3  O  2 O2 ( slow)
= (0.5 - 2x) + 2x + x = 0.5 + x x = Pt - 0.5
Rate = k  O 3   O 2  . Order with respect
2 1

PN 2O5  0.5  2 x = 0.5 - 2(Pt - 0.5) = 1.5 - 2Pt


to oxygen is –1. Total Order is 1.
At t = 100 s; Pt = 0.512 atm

14 CHEMICAL KINETICS
ACTIVE SITE EDUTECH - 9844532971
ACTIVE SITE EDUTECH - 9844532971

Sr |12th class CHEMISTRY VOL-2

PN 2O5 = 1.5 - 2  0.512 = 0.476 atm  A0


In general, amount left after n half lives 
Using equation (3.60) 2n
2.303 P 2.303 0.5 atm Total time
k log i  log
t PA 100 s 0.476 atm No.of half-lives = t1
2
2.303
  0.0216  4.98  104 s 1 Average life of a first order reaction :
100 s Average life of a first order reaction is the time
Half life Period for First order in which the concentration of the reactant is
Reaction reduced to 1/e of its original concentration i.e.,
1st order reaction is given as
 A   A0  / e . Thus,
2.303 a
t log
k ax 1  A0
Here a is the amount of the substance initially Tav  log e
k  A0 / e
present and x is the amount that has reacted in
time ‘t’. In half life time (t1/2), ‘a’ will be 0.693
reduced to a/2 ( x=a/2) 1 1 k 
 log e e  But t1
k k 2
2.303 a
Half-life period (t1/2) = log
k a a/2 t1
 Tav   1.44t 12
2.303 2.303 0.693 2
t1/2 = log 2   0.3010
k k GENERAL INTEGRATED RATE
0.693 EQUATION FOR nth ORDER KINETICS:
t 1/2 
k 1  1 1 
kt =  n 1
 n 1  [n  1]
(n -1)  (a  x) a 
a
If t  t1/ 2 ; x
2

1  2  1 
n 1 n 1

Therefore kt1/ 2       
(n  1)  a   a  

Expression for the amount left after n half 1  2n 1  1 


kt1/2  (n  1) ;
(n  1)  a n 1 
lives
 A0
After one half-life, amount left  1
2 t1/ 2 
a n 1
After two half-lives, amount left
Second Order Reaction:
1  A0  A0 Hydrolysis of ester by alkali (Saponification)
   2
2 2 2
CH3COOC2 H 5  NaOH 
After three half-lives , amount left
CH 3COONa+C2 H 5OH
1  A  A
  20  30
2 2 2 H 2 +I2  2HI

CHEMICAL KINETICS 15
ACTIVE SITE EDUTECH - 9844532971
ACTIVE SITE EDUTECH - 9844532971

Sr |12th class CHEMISTRY VOL-2

2HI  H 2 +I2
2NO2  2NO+O2
2NO2  F2  2NO2 F
NO+O3  NO 2  O 2

2Cl2O  2Cl2  O2
For second order: n = 2
1 x
k 
at  a  x 
Example of fractional order reaction:
1
Half life t1/2  Reaction Order
ak
H 2  Br2  2HBr 1.5
CO  Cl2  COCl2 2.5
COCl2  CO  Cl2 1.5
CH 3CHO  CH 4  CO 1.5
CHCl3  Cl2  CCl 4  HCl 1.5
Pseudo first order Reactions :
There are some reactions in which more than
one species is involved in the rate determining
step, but the order of the reaction is one. Such
reactions are known as pseudo first order
reactions and they involve solvent molecule
Third Order Reactions: or a catalyst as one of the reacting species.
2NO + O2  2NO2 Examples of this type of reactions are:
2NO + Cl2  2NOCl * Ex. 1:- Acid Hydrolysis of an Ester
H
2NO + H 2  N 2O + H 2 O 
CH3COOC2H5+H2O  CH3COOH +
C2H5OH
2FeCl3 + SnCl2  2FeCl2 + SnCl4
for third order n = 3 Rate  k[CH 3COOC2 H 5 ][ H 2O]
Water is in excess then its concentration remain
1  1 1  1 1 1
kt =  2
 2  2
 2 constant during the reaction and [H2O] is taken
(3-1)  (a-x) a  2  (a-x) a  as constant. therefore,
Rate  k '[CH 3COOC2 H 5 ]
where k '  k[ H 2O]
* Ex. 2:- Inversion Of Cane Sugar

H

C12 H 22 O11 + H 2 O 
 C6 H12 O6 + C6 H12 O6
Cane sugar Glucose Fructose

Rate  k[C12 H12O11 ][ H 2O]

16 CHEMICAL KINETICS
ACTIVE SITE EDUTECH - 9844532971
ACTIVE SITE EDUTECH - 9844532971

Sr |12th class CHEMISTRY VOL-2


Water is in excess then its concentration remain Fractional order reactions:
constant during the reaction and [H2O] is taken a) H 2  Br2  2HBr
as constant.
Rate = K H2  Br2 1/ 2 Order  1.5
Rate  k '[C12 H12O11 ] b) CO  Cl2  COCl2
where k '  k[ H 2O]
Rate = K  CO2  Cl2 1/ 2 order  2.5
Ex.3:- Decomposition of benzenediazonium c) COCl2  CO  Cl2
chloride
Rate = K  COCl2 3 / 2 Order  1.5
C6H5N = NCl + H2O  C6H5OH + N2 + HCl
d) CHCl3  Cl2  CCl4  HCl
Isomeric change of N-chloroacetanillide to
p-chloroacetanilide Rate = K CHCl3 Cl2 1/ 2 order = 1.5

COCH3 e) Reaction between H 2 and D2


Cl
C6H 5N  ClC6H 4N
 1/ 2
COCH3 H Rate = K.PH2 .PD2
order = 1.5
f) Conversion of para hydrogen to ortho
Note: For any reaction excess amount of hydrogen at high temperature
reactant is taken then order w.r.t that
Rate   PH 2 
3/ 2
reactant is zero. order = 1.5

a2

CHEMICAL KINETICS 17
ACTIVE SITE EDUTECH - 9844532971
ACTIVE SITE EDUTECH - 9844532971

Sr |12th class CHEMISTRY VOL-2

OTHER IMPORTANT RELATIONS


Zero Reactions:
x1 x2 3
(i) t  t (ii) t75%  t50%
1 2 2
(iii) t100%  2  t50%
First Order Reactions: (b) Plots from integrated rate equations
(i) t75%  2t50% (ii) t87.5%  3t50%
(iii) t93.75%  4t50% (iv) t99.9%  10t50%
10
(v) t90%  t50% (vi) t99%  2t90%
3
(vii) t99.9%  3t90% (viii) t99.99%  4t90%
Second Order Reactions
(i) t2 / 3  2  t1/ 2 (ii) t3 / 4  3  t1/ 2
(iii) t4 / 5  4  t1/ 2 (iv) t99.9%  999t50%
To determine the order w.r.t . each reactant,
follow the following rules:
Change done in conc.
of a reactant Order w.r.t
(Keeping conc. of other Effect on rate
that reactant
reactants constant)
(I) doubled Doubled 1
(ii) doubled/Tripled/
Quadrupled/ Halved, etc No effect 0

(iii) Doubled 4 times 2


(iv) Tripled 9 times 2
(v) Halved Halved 1
(vi) Halved Reduced to ¼ th 2 and so on
Some typical linear plots for the reactions
of different orders :
(a) Plots Of Rate Vs Concentrations
(c) Plots of half-lives vs concentration
Rate  K  conc.
n

( t1/ 2  a1 n )

18 CHEMICAL KINETICS
ACTIVE SITE EDUTECH - 9844532971
ACTIVE SITE EDUTECH - 9844532971

Sr |12th class CHEMISTRY VOL-2


EXAMPLE-14: Calculate the time taken to a= 100 ; x = 20 ; (a – x)
1 = 100 – 20 = 80 ; t = 10 min
reduce th of the original amount if rate
16
constant is given 60s–1. 2.303 100
t 20  log
(1) 4.26 × 10–2s (2) 2.13 × 10–2s k 80

(3) 8.5 × 10–2s (4) 1.065 × 10–2 2.303 100 2.303


10  log ; 10  log 1.25 .....
Sol: Let the initial concentration = a; Final k 80 k
concentration (a – x)=a/16; k=60 s–1. (i)
2.303 a 2.303 IInd part,
t 1
log  log16
(60s ) a /16 (60s 1 ) a = 100, x = 75 (a – x) = 100 – 75 = 25
2.3031.2041 2.303 100 2.303
  0.0462 s  4.26 102 s t75%  log ; t75%  log 4 ..... (ii)
(60 s1 ) k 25 k
EXAMPLE-15: i) Calculate the half life of the Dividing eqn. (ii) by eqn (i),
first order reaction from their rate constant
given as 100 s–1 ( t 75% ) log 4 0.6021
 
10 min log1.25 0.0969
(1)0.693 s (2)6.93 × 10–2s
(3)0.693 × 10–3s (4)6.93 × 10–3s 0.6021
t75% =  10  62.14 min
(ii) Calculate the half life of the first order 0.0969
reaction from their rate constant given as
EXAMPLE-17: Show that the time required for
0.5 min–1.
99.9% completion of the first order
(1) 1.386 min (2) 0.3465 min reaction to take place is 10 times the time
(3) 0.231 min (4) 1.0395 min required for the completion of half of the
Sol: (i)Half life period (t ) = reaction.
1/2
2.303 a
0.693 0.693
  6.93 103 s Sol: For first order reaction, t  log
k ax
k 100s 1
Ist part, a = 100; x = 50;
(ii) Half life period (t1/2) =
(a – x) = 100 –50 = 50
0.693 0.693
  1.386 min. 2.303 100 2.303
k (0.5min 1 ) t1/ 2  log  log 2
k 50 k
EXAMPLE-16: A first order reaction is 20%
2.303  0.3010 0.693
complete in 10 minutes. Calculate the   ..... (i)
k k
period during which 25% of the initial
amount remain. IInd part,
(1) 35 min (2) 20 min a = 100 ; x = 99.9 ; (a – x) = (100 – 99.9) = 0.1
(3) 62.15 min (4) 40 min 2.303 100 2.303
t (99.9%)  log   3 ..... (ii)
Sol: For the first order reaction, k 0.1 k

2.303 a Dividing eqn. (ii) by eqn. (i)


t log
k ax t (99.9%) 2.303  3
  9.97 or 10
Ist part, t (1/ 2) 0.693

CHEMICAL KINETICS 19
ACTIVE SITE EDUTECH - 9844532971
ACTIVE SITE EDUTECH - 9844532971

Sr |12th class CHEMISTRY VOL-2


EXAMPLE-18: A reaction k = 2.2 × 10–5 s–1, t = 90 min
 SO 2  Cl 2 is a first order
SO 2 Cl 2  = 90×60 = 5400 s
reaction with half life 3.15 × 104 s at 2.303 a
320°C. What percentage of SO2Cl2 would (5400 s) = (2.2  105 s 1 ) log a  x
be decomposed on heating at 320°C for
a (5400s)  (2.2  105 s 1 )
90 minutes? log   0.0516
(1) 22.4 % (2) 11.2 % ax 2.303
(3) 5.6 % (4) 2.8 % a
= Antilog 0.0516 = 1.126
0.693 0.693 ax
Sol: k  t  4
 2.2  105 sec1 or a = 1.126a – 1.126x
1/ 2 (3.15  10 s)
0.126
For the first order reaction, t 
2.303
log
a or 0.126 a = 1.126 x or x/a = 1.126  0.112
k ax
or % of SO2Cl2 decomposed
= 0.112×100 = 11.2 %
Zero Order First Order Second Order Third Order

x 1
ax (a  x) 2

t1/ 2 t1/ 2 t1/ 2 t1/ 2

Rectangular hyperbola Hyperbola

t1/ 2 t1/ 2 t1/ 2 t1/ 2

dx dx dx dx
dt dt dt dt

Parabola Cubical Parabola

dx dx dx dx
dt dt dt dt

20 CHEMICAL KINETICS
ACTIVE SITE EDUTECH - 9844532971
ACTIVE SITE EDUTECH - 9844532971

Sr |12th class CHEMISTRY VOL-2


EXAMPLE-19:- 75% of a first order reaction 2.303 0.80
is completed in 30min. Calculate (a) half k log  1.997  103 min 1
30 0.046
life, (b) rate constant and (c) time required
for 99.9% completion of the reaction. The rate constant (k) is the product of pseudo
Sol: Time required for 75% completion is 2 half lifes first order rate constant  k '
= 30min.
and concentration of water. (concentration of

  =15 min
water = 55.5molL1 )
(a) Half-life t 1
2
k  k '  H 2O 
0.693 0.693 1 Substituting the values,
(b) Rate Constant  k     0.046 min
t1 15 1.997 103  k ' 55.5
2
(c) Time required for 99.9% of the reaction Pseudo rate constant = k '
 3.6 105 L mol 1 min 1
2.303 a 2.303 100 EXAMPLE-22: For a reaction
 t 
log log 149min
k a  x 0.046 0.1 A+2B  products, when B is taken in
excess, then the rate law expression and
EXAMPLE-20: A first order reaction is 20%
order is
completed in 10min. How long it takes to
Sol: For the reaction rate law expression is
complete 80%?
Rate=k[A]1[B]0  Order =1
Sol: Applying first order integral rate equation, rate
constant k is given as, EXAMPLE-23: The following data were
obtained during the first order thermal
2.303 a 2.303 100
k log  log decomposition of N 2O5  g  at constant
t ax 10 100  20
volume :
 0.0223min 1
2 N 2O5  g   2 N 2O4  g   O2  g 
 log1.25  0.0969  Time required for 80% S.NO Time(s) Total Pressure/(atm)
1. 0 0.5
completion of the first order reaction, t0.8 2. 100 0.512
Calculate the rate constant.
2.303 100 2.303 100
t0.8  log  log  72.2 min Sol: Let the pressure of N 2O5  g  decreases by 2x
k 100  80 0.0223 20
EXAMPLE-21:- The initial concentration of atm. As two moles of N 2O5 decompose to
ethyl acetate is 0.85molL1 . Following the give two moles of N 2O4  g  and one mole of
acid catalysed hydrolysis, the
O2  g  , the pressure of N 2O4  g  increase by
conentration of ester after 30min and
60min of the reaction are respectively 0.8 2x atm and that of O2  g  increases by x atm.
and 0.754molL1 . Calculate the rate 2 N 2O5  g   2 N 2O4  g   O2  g 
constant and pseudo rate constant. Start t  0 0.5 atm 0atm 0atm
Sol: Acid catalysed ester follows pseudo first order At timet  0.5  2x  atm 2x atm x atm
kinetics. The rate constant k is given as
Pt  PN2O5  PN2O4  PO2
2.303 a
k log   0.5  2 x   2 x  x  0.5  x
t ax
x  Pt  0.5 PN2O5  0.5  2 x

CHEMICAL KINETICS 21
ACTIVE SITE EDUTECH - 9844532971
ACTIVE SITE EDUTECH - 9844532971

Sr |12th class CHEMISTRY VOL-2

 0.5  2  Pt  0.5   1.5  2 Pt  B2  (0.6)2  1.2


At t  100 s; Pt  0.512 atm  A (0.3)
Step III: Calculation of initial rate of
PN 2O5  1.5  2  0.512  0.476 atm conversion of A.
2.303 P 2.303 0.5 0.2 M
But k  log i  log A  0.1 M hr 1
t Pf 100 0.476 2 hr

2.303 EXAMPLE-25: A first order reaction is 50%


  0.0216  4.98  104 s 1 completed in 30 minutes at 27°C and in 10
100
minutes at 47°C. Calculate the reaction rate
EXAMPLE-24: The progress of the reaction
constants at these temperatures and the
 nB ,with time is presented in the
A  energy of activation of the reaction in kJ/
figure. Determine (i) the value of n, (ii) mol
the equilibrium constant, k, and (iii) the (R= 8.314 J mol–1 K–1)
initial rate of conversion of A. (1) 44 J (2) 88 J (3) 44 kJ(4) 88 kJ
Sol:
Sol: The half life period for first order reaction is
0.693 0.693
given by : t1/ 2  k
or k 
t1/ 2

0.693
Therefore, k1   0.0231min 1
30 min

0.693
and k 2  or k 2  0.0693min 1
10 min
According to Arrhenius equation,
Step I: Calculation of the value of n,From the
curves Decrease in concentration of A in 4 k2 Ea  1 1 
log    
hours (1 to 5) = 0.5 – 0.3 = 0.2 mol. k1 2.303R  T1 T2 
Increase in concentration of A in 4 hours (1 to Here, k1 = 0.0231 min–1; k2 = 0.0693 min–1;
5) T1 = 27 + 273 = 300 K; T2 = 47 + 273 = 320
= 0.6 – 0.2 = 0.4 mol K
d[A] 0.2 d[B] 0.4
 and  (0.0693 min 1 )
dt 4 dt 4 log =
(0.0231 min 1 )
The reaction rate (r) may be given as :
Ea  1 1 
d[A] 1 d[B]  0.2  1  0.4  
 or       1 1  
dt n dt  4  n 4  2.303  (8.314 J mol K )  300 K 320 K 

0.4 Ea 20
n=– 2  log 3 = 2.303  8.314(J mol1 )  300  320
0.2
Step II: Calculation of Equilibrium constant
Ea
The equation may now be written as :  0.4771 = 2.303  8.314(J mol1 )  4800
 2B
A 
 Ea = 0.4771 × 2.303 × 8.314 × 4800 (J
 Equilibrium constant (k) = mol–1)
 Ea = 43848 J mol–1 = 43.848 kJ mol–1

22 CHEMICAL KINETICS
ACTIVE SITE EDUTECH - 9844532971
ACTIVE SITE EDUTECH - 9844532971

Sr |12th class CHEMISTRY VOL-2


EXAMPLE-26: At 380°C the half-life period for [A]0
thefirst order decomposition of H2O2 is 360 = Antilog 0.1172 = 1.310
[A]t
min. The energy of activation of the
reaction is 200 kJ mol–1. Calculation the [A]0 1M
time required for 75% decomposition at [ A ]t    0.763 M
1.310 1.310
450°C ?
(1) 20.35 min (2) 40.7 min Step II. Calculation of rate after 1 h (60 min).
(3) 120 min (4) 60 min Rate after 60 min = k[A]t = 4.5×10–3
Sol: Step I: Calculation of rate constant (k1) at
380°C min–1 ×0.763 M = 3.4354×10–3 M min–1.
0.693 0.693 Methods of Determination of order of reaction:
k1   min 1
t1/ 2 360 1) Trail and Error method or Integrated form
Step II:Calculation of rate constant (k2) at of rate equation method
450°C
Ea = 200 × 103 J mol–1;
T1 = 380°C + 273 = 653K;
T2 = 450°C + 273 = 723 K Zero order R  P0
k2 E a  T2  T1 
log 
k1 2.303 T1T2
x a  a  x
x  kt (or ) k 
k2 t t
k1 = Antilog 1.5487 = 35.38 First order R  P
0.693 2.303 a
k2 = 35.38 × k1 = 35.38 × =0.0681 k log
360 t a  x
min–1
Step III: Calculation of time required for 75%
2.303
decomposition 0.0681min 1  0.6021  20.35 min
EXAMPLE-27: The rate constant for an
isomerization reaction A  B is4.5×10–3min–
1
. If the initial concentration of A is 1M.
Calculate the rate of the reaction after 1 h.
(1) 3.45 × 10–3 M min–1
(2) 6.9 × 10–3 M min–1
(3) 1.73 × 10–3 M min–1
(4) 1.5 × 10–3 M min–1
Sol:
Step I:Calculation of concentration after1h
(60min).

k
2.303 A
log 0 Second order 2R  P
t [A]t
1 x
k 
log
[A ]0

kt

4.5  10 3 min 1 60 min
 0.11729 at  a  x 
[ A ]t 2.303 2.303

CHEMICAL KINETICS 23
ACTIVE SITE EDUTECH - 9844532971
ACTIVE SITE EDUTECH - 9844532971

Sr |12th class CHEMISTRY VOL-2


EXAMPLE 28: The reaction :
 4NO 2 (g)  O 2 (g) was
2N 2 O5 (g) 
studied and the following data were
collected

Second order R1  R2  P
2.303 b a  x
k log
t a  b a b  x 

Determine
(i) the order,
(ii) the rate law and
(iii) rate constant for the reaction.
Sol: Let the order of the reaction = n
2) Half-time  t1/ 2  method
Rate  [N2O5]n
n 1
1  t   a' ' 
'

t1/ 2  n1 ;  ' '    ' 


1/2
(34 × 10–5)  (1.13 × 10–2)n...... (i)
a  t1/ 2   a 
(25 × 10–5)  (0.84 × 10–2)n...... (ii)
where n = order of reaction
3) Van’t Hoff Differential method Dividing eqn. (i) by eqn. (ii), we have
dC
 kC n (34  10 5 ) (1.13  10 2 ) n
dt 
(25  10 5 ) (0.84  10  2 ) n
For two initial concentrations C1 , C2 we have
dC1 dC2 (1.36)1 = (1.35)n
 kC1n ;  kC2n
dt dt n = 1
 dC1   dC2  (i)The order of the reaction = 1
log    log  
 dt   dt 
n  (ii)Rate law : The rate of reaction depends upon
 log C1  log C2  one concentration term of N2O5 because
4) Ostwald’s Isolation method order of the reaction is 1.
This method is useful to determine the order
w.r.t each reactant of a reaction separately by  Reaction Rate = k[N2O5]
taking other reactants in excess quantity.
Reaction rate
A + B + C  products (iii)Rate constant k =
[N 2 O5 ]
Then order with respect to A is n A
Order with respect to B is n B 18  10 5 mol L1 min 1
 2 1
 0.029 min 1
Order with respect to C is n C 0.62  10 mol L

overall order of the reaction = n A  n B  n C

24 CHEMICAL KINETICS
ACTIVE SITE EDUTECH - 9844532971
ACTIVE SITE EDUTECH - 9844532971

Sr |12th class CHEMISTRY VOL-2


EXAMPLE-29: In a reaction between A and Collision Theory of reaction rates or
B, the initial rate of reaction was measured Kinetic Molecular theory:
for different initial concentration of A and  Collision theory was proposed by Arrhenius
B as given below : and developed by Max Trautz and William
Lewis.
 The main postulates of collision theory are
a) Collisions must occur between the
molecules of reacting gases for a reaction
to occur.
b) All collisions do not lead to the formation
of products. (Only fruitful collisions leads
to formation of products)
c) The minimum amount of energy possessed
by the colliding molecules to the formation
What is the order of reaction with respect of products or reaction to occur is known
to A and B? as threshold energy.
1 1 d) The energy possessed by the molecules at
(1)1, 2 (2)2, 1 (3) , 0 (4) , 1 STP is known as normal energy or internal
2 2
energy.
Sol: The rate law equation may be expressed as:
e) Normal energy possessed by normal
ate = k[A]p [B]q ;
molecules is always less than threshold
Comparing experiments 1 and 2 energy.
f) The minimum additional energy to be
(Rate)1 = k [0.2]p [0.3]q = 5.07 × 10–5...... (i)
gained by the normal molecules during the
(Rate)2 = k [0.2]p [0.1]q = 5.07 × 10–5..... (ii) collision to convert into products is known
as activation energy or energy of activation.
Dividing eqn. (i) by (ii)
g) Activation energy =
( Rate)1 k[0.2]p [0.3]q 5.07  10 5
Threshold energy - energy of normal
  molecules
( Rate) 2 k[0.2]p [0.1]q 5.07  10 5
h) Activation energy increases, the rate of the
[3]q = [1]0 or q = 0 reaction decreases.
i) No. of binary collisions per unit time (Z) is
By experiment no. 3
8kT
Rate  ZAB e  Ea / RT Z   AB
2
(Rate)3 = k[0.4]p [0.05]q = 7.16 × 10–5 * nA nB ;

..... (iii)
Dividing eqn. (iii) by eqn. (i)  AB  collision diameter ;   reduced mass

p q 5
j) Specific rate k   Z e  Ea / RT or
( Rate) 3 k[0.4] [0.5] 7.16  10
 
p
( Rate) 2 k[0.2] [0.1] q
5.07  10 5 k  Ae  Ea / RT Where  = probability
factor
[2]p  [1.412]  [2]1/ 2 or p  1 / 2 The constant ‘A’ has unit of time–1 and is
constant for a given reaction
Order with respect to A = 1/2 At very high temperature rate becomes
Order with respect to B = 0 equal to frequency factor, i.e., k = A.
 Activation energy of HI decomposition reaction
is 52.8 kJ/mole.
CHEMICAL KINETICS 25
ACTIVE SITE EDUTECH - 9844532971
ACTIVE SITE EDUTECH - 9844532971

Sr |12th class CHEMISTRY VOL-2


 For 2NO2  2NO+O2 the activation energy
is 111kJ/mole. So decomposition of NO2(g) is
slower than the decomposition of HI(g).
* The collisions in which molecules collide with
sufficient kinetic energy (called threshold energy)
and proper orientation, so as to facilitate
breaking of bonds between reacting species
and formation of new bonds to form products
are called effective collisions. Where as
improper orientation makes them simply bounce
back and no products are formed.
* For example, formation of methanol from
bromomethane.

 The fraction of activated collisions is smaller If activation energy of forward reaction  Eaf 
than the total number of collisions.
is less than that of the backward
 Actual rate of reaction is much smaller than the
rate of the reaction calculated on the basis of reaction  Eab  , the reaction is exothermic.
the normal collisions.
The heat of the reaction, H  Eaf  Eab
Concept of Activation Energy: The
difference between the energy barrier (i.e., Reactions with lower activation energy are fast
and with higher activation energy are slow.
threshold energy) ET and the energy of normal
For ionic reactions, the energy of activation is
molecules ER is called activation energy Ea . negligibly small and hence they are
Ea  ET  ER instantaneous.
For covalent reactions, the energy of activation
is high and the reactions are time consuming.
In the presence of a catalyst the path is altered,
with a new path of low activation energy, the
time required for a covalent reaction is also low.
Increasing the concentration of reactants
increases the rate. This is because of the
increase in the collision frequency and increase
in number of reactant molecules crossing the
energy barrier.

26 CHEMICAL KINETICS
ACTIVE SITE EDUTECH - 9844532971
ACTIVE SITE EDUTECH - 9844532971

Sr |12th class CHEMISTRY VOL-2

Above graph is showing plot of potential energy Vs


Collision Frequency (Z) : Total number of
reaction coordinate.
collisions which occur among the reacting
molecules per second per unit volume is called Photochemical reaction: Reactions which
collision frequency. Its value is given by take place by the absorption of radiations of
suitable wavelength
Z  2  v 2 n 2
Ex :- H 2  g   Cl2  g  
light
 2HCl  g 
v = average velocity
Photosynthesis of carbohydrates in plants takes
 = molecular diameter in cm
place in presence of chlorophyll and sunlight
n = number of molecules per cc.
6CO 2  6H 2 O 
light
 C 6 H12 O 6  6O 2
 Rate  k   ze
 Ea / RT
The free energy change of a photochemical
Transition State theory: According to this reaction may not be negative.
theory, the bimolecular reaction between two In the synthesis of carbohydrates and formation
molecules A 2 and B2 passes through the of HCl. G is +ve.
formation of activated complex which then Quantum Efficiency OR Quantum yield
decomposes to yield the product AB, as shown Number of molecules participating in
below photochemical reaction with absorption of
A 2  B2   A 2 B2   2AB
quanta is called Quantum Efficiency. It is
Re ac tan ts  Activated Complex 
Pr oduct
expressed as
Ex: H 2 g   I 2 g   2 HI  g  
Number of molecules reacting in a given time
Number of quanta of light absorbed in the same time
According to Arrhenius, this reaction can take
place only when a molecule of hydrogen and a Chemiluminescence: Emission of light in a
molecule of iodine collide to form an unstable chemical reaction at ordinary temperatures is
called chemiluminescence.
intermediate. It exists for a very short time and
Fluorescence: The absorption of energy and
then break up to form two molecules of
instantaneous reemitting of the energy is called
hydrogen iodide.
fluorescence.
Phosphorescence : The continuous glow of
some substances even after the cutting of source
of light is called phosphorescence. Eg. ZnS
Bioluminescence : The phenomenon of
(transition state) chemiluminescence exhibited by certain living
organisms is called Bioluminescence
Eg. light emission by fire flies.
ADDITIONAL INFORMATION
 Determination of rate constant in some
important types of First Order Reactions
Decomposition of N2O5 : It follows first order
kinetics. The compound, N2O5, is a volatile
solid which decomposes in the gaseous state
as well as in the form of its solution in an inert
solvent like carbon tetrachloride, chloroform
etc. according to the equation:

CHEMICAL KINETICS 27
ACTIVE SITE EDUTECH - 9844532971
ACTIVE SITE EDUTECH - 9844532971

Sr |12th class CHEMISTRY VOL-2


1
N2O5 

 N2O4 + 2 O2 2.303 162
(iii) K  log  3.26  103 min 1
70 162  33
 The fairly constant values of K. indicate
2NO2 that the reaction is of the first order.
Decomposition of H2O2: The decomposition
When the reaction is carried out in the solution,
of hydrogen peroxide, is found to be of first
N2O4 and NO2 remain in the solution and the
order. The decomposition of hydrogen
volume of oxygen gas collected is noted at peroxide in aqueous solution (catalysed by the
different intervals of time. It is obvious that presence of finely divided platinum) takes place
Volume of oxygen gas collected at any time (Vt) according to the equation:
 Amount of N2O5 decomposed (x) 1
i.e., x Vt H2O2 
 H2O + O2
2
Volume of oxygen gas collected at infinite time The kinetics of this reaction may be studied
(V  )  Amount of N2O5 initially taken (a) either by collecting the oxygen gas produced
i.e., a V and noting its volume at different intervals of
time or by making use of the fact that H2O2
Substituting these values in the first order solution can be titrated against KMnO4
equation. solution. Thus by withdrawing equal amounts
of the solution (usually 5 cc) at regular intervals
2.303 a
k log ; of time and titrating against the same KMnO4
t ax
solution, the amount of H2O2 present can be
2.303 V found every time. It is obvious that for the same
k log
t V  Vt volume of the reaction solution withdrawn:
Volume of KMnO4 solution used at
The constancy in the value of k for given set of t = 0 i.e V0  initial amount of H2O2
experimental data proves the reaction tobe first (a) i.e., a  V0
order. Volume of KMnO4 solution used at any time
EXAMPLE 30:Decomposition of benzene i.e Vt amount of H2O2 present at that time
diazonium chloride was studied at a (a - x) i.e., a - x  Vt
constant temperature by measuring the Substituting these values in the first order
volume of N 2 gas evolved at different equation
intervals of time. Using the following data 2.303 a
k log we get
show that the reaction is of the first order t ax
Time in minutes: 0 20 50 70  2.303 V
k log 0
Vol. of N2 t Vt
measured in mL: 0 10 25 33 162 Example 31: From the following data estimate
Sol: the order for decomposition of an aqueous
When t = 20, solution of hydrogen peroxide:
V  Vt   162  10  152 Time (minutes) 0 10 20
30
2.303 162
(i) K  log  3.22  103 min 1 V (ml) 46.1 29.8 19.6
20 162  10 12.3
2.303 162 where V is the volume of potassium
(ii) K  log  3.36  103 min 1
50 162  25 permanganate solution in ml required to
28 CHEMICAL KINETICS
ACTIVE SITE EDUTECH - 9844532971
ACTIVE SITE EDUTECH - 9844532971

Sr |12th class CHEMISTRY VOL-2


decompose a definite volume of the As the value of K are constant, the given
peroxide solution. isomeric reaction is of the first order.
(A) 0 (B) 1 (C) 2 (D) 1/2 Example 33: The gaseous reaction,

2.303 V A(g)   2B (g) + C(g) is observed to


Sol: (B) k1  t
log 0 , we get
Vt be the first order. On starting with pure
A, it is found that at the end of 10 min, the
2.303 46.1 total pressure of the system is 176 mm of
(i) k1  log  4.364 102
10 29.8 Hg and after a long time, it is 270 mm of
Hg. Which of the following is /are correct
2.303 46.1 for the given data
(ii) k1  log  4.276 102
20 19.6 (A) the initial pressure A is 90 mm Hg
2.303 46.1 (B) the partial pressure of A after 10 min
(iii) k1  log  4.404  102
30 12.3 is 47 mm Hg
Since k1 comes out to be nearly constant, the (C) the rate constant of the reaction is
reaction is a first order one with k as the average 0.0649 min-1
value of the three. (D) all are incorrect
EXAMPLE 32: From the following data of the Sol: (A, B, C) Let the initial pressure of A be P0
isomeric change of N-chloroacetanilide mm of Hg and the pressure of A decreases in
into p-chloracetanilide in acidic medium, 10 minute be x unit.
show that the isomeric change is a reaction A(g) 
 2B (g) + C (g)
of first order.
Initially P0 0 0
Time in hrs: 0 1 2 3
Vol of hypo used in mL:49.3 35.6 25.73 18.50 At time 10 min. P0 – x 2x x
Sol: The first order rate constant for the At  time 0 2P0 P0
conversion of N-chloroacetanilide into p- After long time interval, P  = 2P0 + P0 = 3P0
chaloroacetanilide is given by
P
 P0   90 mm of Hg
2.303 V 3
K log 0
t V1
After 10 minutes, Pt=P0+2x
In this problem V0  49.3 at zero time. 176 = 90 +2x
Substituting the values, we get  x = 43 mm of Hg
2.303 49.3  Partial pressure of A after 10 min = P0 – x =
(i) K  log  3.25  101 hr 1 90 – 43 = 47 mm of Hg.
1 35.6
For a first order reaction, the rate constant
2.303 49.3
(ii) K  log  3.25  101 hr 1
2 25.75 2.303 P
expression would be k= log 0
t P0 - x
2.303 49.3
(iii) K  log  3.27  101 hr 1
3 18.50 2.303 90
k= log = 0.0649 min-1
10 47

CHEMICAL KINETICS 29
ACTIVE SITE EDUTECH - 9844532971
ACTIVE SITE EDUTECH - 9844532971

Sr |12th class CHEMISTRY VOL-2


 Hydrolysis of Ethyl acetate : Time (minutes) 0 10 20 30 60 

CH3COOC2H5+H2O H CH3COOH+ Rotation (degrees) +32 25.5 20.0 15.5 5.0 -10.50
C2H5OH (A) 0.0166 min-1 (B) 0.0164 min -1
at Initial stage 0 0 (C) 0.0168 min-1 (D) none of these
at time t a- x x Sol: (A) Here a, the initial concentration  (r0 –
after long time 0 aa r )
In this reaction acetic acid is one of the x, the change in time t  (r0 – rt)
products, the amount of which can be calculate
 (a –x)  (rt – r  )
by titrating against standard NaOH solution.
Being an acid-catalysed reaction, the acid Substituting these values for different (a – x)
present originally as catalyst, also reacts with values corresponding to time t from the data in
NaOH solution. Hence to calculate the amount the equation for first order reaction
of acetic acid the reaction mixture withdrawn
2.303 a
at different time interval is titrated with NaOH. k log =
t ax
So, volume of NaOH solution used at begining
i.e. V0  Amount of acid present as catalyst 2.303 r r
volume of NaOH solution used at any time t log 0  , we get
t rt  r
i.e. Vt  Amount of acid presen++t as catalyst
+ amount CH3COOH produced (x) 2.303 32  (10.5)
 CH3COOH produced at any time i.e. x  i) k1  log 
10 25.5  (10.5)
(Vt - V0)
Volume of NaOH solution used after long time 2.303 42.5
log  0.0166 min -1
 i.e. V   Amount of acid present as 10 36.0
catalyst + maximum amount of CH3COOH
produced (a) 2.303 32  (10.5)
ii) k1  log 
 maximum amount of CH3COOH produced 20 20.0  (10.5)
(a)  (V  - V0)
(a – x)  (V - V0 ) - (Vt – V0) 2.303 42.5
log  0.0166 min -1
20 30.5
or (a – x)  (V - Vt )
2.303 32  (10.5)
Substituting the values of a and (a – x) in the iii) k1  log 
first order equation, we get 30 15.5  (10.5)

2.303 a 2.303 42.5


k= log or log  0.0164 min -1
t a-x 30 26.0
2.303 V -V
k= log  0 2.303 32  (10.5)
t V - Vt iv) k1  log 
60 5.0  (10.5)
EXAMPLE 34: The optical rotation of sucrose
in 0.5 N-hydrochloric acid at 308 K and 2.303 42.5
log  0.0168 min -1
at various time intervals are given below. 60 15.5
The rate constant for the first order
Average value of k1 = 0.0166 min–1
hydrolysis of sucrose will be

30 CHEMICAL KINETICS
ACTIVE SITE EDUTECH - 9844532971
ACTIVE SITE EDUTECH - 9844532971

Sr |12th class CHEMISTRY VOL-2


6. The rate of a chemical reaction generally
increases rapidly even for small
temperature increase because of rapid
increase in the
(1) Collision frequency
RATE OF REACTION & FACTORS (2) Fraction of molecules
1. For the reaction, 4A+B  2C+2D, The (3) Activation energy
statement not correct is : (4) Average kinetic energy of molecules
(1) The rate of disppearance of B is one fourth 7. The pre-exponential factor in the Arrhenius
the rate of disappearance of A equation of a second order reaction has the
(2) The rate of appearance of C is half the rate units
of disappearance of B (1) mol L–1 s–1 (2) L mol–1 s–1
(3) The rate of formation of D is half the rate (3) s–1 (4) dimensionless
of consumption of A 8. The rate of most of the reactions double
(4) The rates of formation of C and D are equal when the temperature is raised from 298
2. Which of the following statement is correct K to 308 K. Calculate their activation
for a reaction X + 2Y  prodcuts : energy.
(1) The rate of disappearance of X = twice (1) 52.89 kJ (2) 53 J
the rate of disappearance of Y (3) 106 kJ (4) 106 J
(2) The rate of disappearance of X = 1/2 rate 9. For a reaction, the activation energy is
of appearance of products zero. What is the value of rate constant at
(3) The rate of appearance of products = 1/2 300 K if
the rate of disappearance of Y k=1.6 × 106 s –1 at 280 K (R=8.31JK–1mol– 1)?
(4) The rate of appearance of products = 1/2
the rate of disappearance of X (1)  (2) 0
3. The rate law for the single-step reaction , (3) 1.6 × 106 s (4) 8.31 s–1
2A + B  2C , is given by : 10. Under a given set of experimental
conditions with increase in the
(1) Rate = k[A] [B] (2) Rate = k[A]2 [B]
concentration of the reactants, the rate of
(3) Rate = k[2A] [B] (4) Rate = k[A]2 [B]0
chemical reaction
4. For the reaction 2NO 2 +F 2  2NO 2 F,
1) decreases 2) increases 3) remains constant
following mechanism has been provided :
4) first decreases and increases
NO2 + F2  NO2F + F (slow)
11. In a chemical reaction, rate of a chemical
NO2 + F  NO2F reaction increases with temperature. The
Thus rate expression of the above reaction reason is due to
can be written as : 1) number of collisions between molecules
(1) r = k [NO2]2 [F2] (2) r = k [NO2][F2] increases
(3) r = k [NO2] (4) r = k [F2] 2) decreases in activation energy
5. Following mechanism has been proposed 3) increase in the number of the molecules with
for a reaction , 2 A + B  D + E activation energy
A + B C + D . . . . . . . . (slow); 4) kinetic energy of reactants increases
12. Which of the following reaction is a fast
A + C  E ...... (fast) reaction at laboratory temperature
The rate law expression for the reaction 1) reaction between KMnO4 and oxalic acid
is: 2) reaction between KMnO4 and mohr's salt
(1) r = k[A]2[B] (2) r = k [A] [B] 3) hydrolysis of ethyl acetate
(3) r = k[A]2 (4) r = k[A] [C] 4) thermal decomposition of N2O5
CHEMICAL KINETICS 31
ACTIVE SITE EDUTECH - 9844532971
ACTIVE SITE EDUTECH - 9844532971

Sr |12th class CHEMISTRY VOL-2


13. k represents the rate constant of a reaction 20. Chemical kinetics is a branch of physical
when log k is plotted against 1/T chemistry deals with
(T=temperature) the graph obtained is a 1) structure of molecules
1) curve
2) heat changes in a reaction
2) a straight line with a constant positive slope
3) a straight line with constant negative slope 3) physical changes in a reaction
4) a straight line with no slope 4) rate of reactions
14. Slowest reaction among the following 21. The rate of a reaction
under identical conditions is 1) increase with increase in temperature
1) NaOH  HCl  NaCl  H 2O 2) decrease with increase in temperature
2) H   OH   H 2O 3) does not depend on temperature
3) 2 NO  O2  2 NO2 4) does not depend on concentration
22. The rate of chemical reaction would
4) CH 4  2O2  CO2  2 H 2O
1) increase as the reaction proceeds
15. In reactions involving gaseous reactants
and gaseous products the units of rate are 2) decrease as the reaction proceeds
1) Atm 2) Atm-sec 3) may increase or decrease during the reaction
3) Atm.sec -1
4) Atm sec
2 2 4) remains constant as the reaction proceeds
16. In the sequence of reaction 23. The factor which does not influence the rate
A  k1
 B  k2
 C  k3
D k 3>k 2 >k 1 of reaction is
then the rate determining step of the 1) Nature of reactants
reaction is 2) Concentration of the reactants
1) A  B 2) C  D 3) B  C 4) A  D 3) Temperature 4) Molecular mass
17. A graph is drawn between the
24. The rate at which a substance reacts
concentration of the reactants (taken on
depends upon its
y-axis) and the time of reaction (taken on
x-axis). The slope of the tangent drawn to 1) Active mass 2) molecular mass
the graph at a point corresponding to 3) atomic mass 4) equivalent mass
reaction time t sec. gives 25. The term dc/dt in a rate equation refers to
1) rate of the reaction 1) concentration of reactants
2) rate constant of the reaction
2) change in concentration of reactants or
3) rate of reaction at time t products with time
4) half life period of the reaction
3) velocity constant of the reaction
18. In which of the following cases, does the
reaction goes farthest to completion 4) concentration of products
1) k=102 2) k=10-2 3) k=10 4) k=1 26. The rate of chemical reaction depends on
19. At 298 K, 1 atm, among the nature of reactants because
A) 2 H 2  O2  2 H 2O 1) The number of bonds broken in the reactant
molecules and the number of bonds formed
B) H 2  Cl2  2 HCl in product molecules changes
C) N 2  O2  2 NO 2) Some of the reactants are solids at the room
D) H 2 SO4  KOH  products, correct temperature
order of reaction rates is 3) Some of the reactants are coloured
1) D>A>C>B 2) D<A<B<C 4) Some of reactants are liquids at room
3) D>B>A>C 4) D>B=C>A temperature

32 CHEMICAL KINETICS
ACTIVE SITE EDUTECH - 9844532971
ACTIVE SITE EDUTECH - 9844532971

Sr |12th class CHEMISTRY VOL-2


27. The relation between the rate of a simple 3) The average rates of increase in the
reaction and the concentration 'c' of the concentration of NO and O2 are expressed
reacting species is given as d N O  d O 2 
as and
1 dt dt
1) rate  c 2) ra te 
c d  NO  2 d  O2 
4) 
dt dt
1
3) rate  34. The rate constant is given by the equation
cn
k  A.e  Ea / RT which factor should register
4) rate  c n (n=order of reaction)
a decrease for the reaction to proceed more
28. Dimensions of rate of reaction involves rapidly
1) concentration only
1) T 2) k 3) A 4) Ea
2) time only
3) both concentration and time 35. Arrhenius equation may be written as
4) neither time nor concentration d ln k Ea d ln k E
29. Which of the following about the rate 1)  2)  a2
constant k of a reaction is wrong ? dT RT dT RT
1) it remains unchanged throughout the course d ln k E d ln k E
of reaction 3)  a 4)   a2
dT RT dT RT
2) it provides a convenient measure of reaction
rate 36. In Arrhenius plot, intercept is equal to
3) it is expressed in the same unit(sec)-1 for all Ea
reactions 1)  2) ln A 3) ln k 4) log10 a
R
4) the more rapid the reaction, the larger is
37. The rate of gaseous reaction is given by
the value of k, the slower the reaction the
k[A] [B]. If the volume of reaction vessel is
smaller is its value
30. The value of the rate constant of a reaction 1
reduced to of initial volume the reaction
depends on 4
1) time 2) activation energy rate relative to the original rate is
3) temperature 4) half-life value 1 1
31. For an irreversible chemical reaction, the 1) 2) 3) 8 4) 16
16 8
concentration of the products with time 38. The rate of reaction for A  products is 10
1) increases
2) decreases mole.lit -1 .min -1 when t1=2min. The rate of
3) does not change reaction when t2=12min. in the same units
4) some more data required is
32. A catalyst 1) >10 2) <10
1) Increases the heat of the reaction 3) 10 4) 12
2) Decreases the heat of the reaction
39. C12 H 22O11  H 2O  C6 H12O6  C6 H12O6
3) Does not alter the heat of the reaction
(excess) ( glu cos e) ( fructose)
4) Increases the number of collisions
33. For the reaction 2NO2  2NO+O2 which Rate law is expressed as
of the following is false ? 1) r  k C12 H 22O11  H 2O 
1) The decrease in [NO2] and the increase in
[NO] proceed at the same rate 2) r  k C12 H 22 O11  3) r  k  H 2 O 
2) The rate of formation of NO is twice the
4) r  k C12 H 22 O11  H 2 O 
2
rate of formation of O2
CHEMICAL KINETICS 33
ACTIVE SITE EDUTECH - 9844532971
ACTIVE SITE EDUTECH - 9844532971

Sr |12th class CHEMISTRY VOL-2


40. A chemical reaction was carried at 300K 45. For the reaction
and 280K. The rate constants were found 4NH 3  5O 2  4NO  6H 2 O, the rate of
to be k 1 and k 2 respectively. then reaction with respect to NH is 3
1) k 2  4 k 1 2) k 2  2 k 1
2  10 Ms . Then the rate of the
3 1

3) k 2  0.25 k 1 4) k 2  0.5 k 1 reaction with respect to oxygen in Ms 1


41. The differential rate law for the reaction
1) 2  103 2) 1.5  10 3
H 2  I 2  2 HI is
3) 2.5  10 3 4) 3  103
d  H 2  d  I 2   d  HI 
1)   46. The rate of formation of SO3 in the reaction
dt dt dt
2SO 2  O 2  2SO3 is 100g min 1 . Hence
d H2  d  I2  d  HI 
2)  
dt dt dt rate of disappearance of O2 is
1 d  H 2  d  I 2  d  HI  1) 50g min 1 2) 100g min 1
3)  
2 dt dt dt 3) 20 g min 1 4) 40 g min 1
d H2  d  I2  d  HI  47. The rate of the reaction at 400C is 5 units,
4) 2  2 
dt dt dt then the rate of same reaction at 800C is
42. Observe the following reaction (nearly)
A g   3B g   2C g  . The rate of this 1) 10 units 2) 40 units
3) 20 units 4) 80 units
 d  A  48. Consider the following reaction
reaction    is
 dt  N 2 ( g )  3H 2 ( g )  2 NH 3 ( g )
3×10-3 mole lit -1 min -1 . What is the The rate of this reaction in terms of N2 at
d B  d N 2 
value of - in mole lit -1 min -1 ? T(K) is  0.02 m ole .lit  1 .sec  1
dt dt
1) 3  10 3 2) 9  103  d H 2 
what is the value of (in units of
3) 103 4) 1.5  10 3 dt
43. For which of the following reactions mole lit-1 sec-1) at the same temperature.
k310 / k300 would be maximum 1) 0.02 2) 50
3) 0.06 4) 0.04
1) A  B  C ; Ea  50 kJ
49. Rate Equation Units of k
2) X  Y  Z ; Ea  40 kJ I) rate = k[A] a) mole lit-1 sec-1
3) P  Q  R ; Ea  60 kJ II) rate = k[A] [B] b) lit2 mole-2 sec-1
4) E  F  G ; Ea  100 kJ III) rate = k[A] [B]2 c) sec-1
44. The slope in the activation energy curve IV) rate = k d) lit mole-1 sec-1
is 5.42  103 . The value of the activation The correct matching is
energy is approximately 1) I - d, II - c, III - a, IV - b
1) 104 J mol1 2) 104 MJ mol1 2) I - c, II - d, III - b, IV - a
3) I - a, II - b, III - c, IV - d
3) 104 kJ mol 1 4) 104 J mol1 K 1 4) I - b, II - a, III - d, IV - c

34 CHEMICAL KINETICS
ACTIVE SITE EDUTECH - 9844532971
ACTIVE SITE EDUTECH - 9844532971

Sr |12th class CHEMISTRY VOL-2


50. Assertion (A) : Rate of reaction will be 55. In the process 2 N 2O5( g )  4 NO2( g )  O2( g )
doubled, when temperature increased from
298 K to 308 K. at t = 10 rate of reaction w.r.t N 2O5 ,
Reason (R) : The activation energy of NO2 & O2 respectively are
reaction decreases with increase in
temperature. N 2O5 NO2 O2
1) Both (A) and (R) are true (R) is the correct 1) 500mm/min 400mm/min 2 0 0 m m / min
explanation to (A)
2) Both (A) and (R) are true but (R) is not
the correct explanation to (A) 2) 1000mm/min 1000mm/min 5 0 0 m m / min
3) (A)is true but (R) is false
4) Both (A) and (R) are false 3) 1000mm/min 2000mm/min 4000mm/ min
51. For the elementary reaction 2A  C the
concentration of A after 30 minutes was 4) 400mm/min 400mm/min 4 0 0 m m / min
found to be 0.01 mole/lit. If the rate
constant of the reaction is 2.5 × 10-2 lit mole- 56. The rate of the reaction
1
sec-1 the rate of the reaction at 30 minutes
is CH3COOC2H5 +NaOH  CH3COONa +C2H5OH
1) 2.5x10-4 mole lit-1 sec-1 is given as rate =k[CH 3 COOC 2 H 5 ]
2) 2.5x10-6 mole lit-1 sec-1 [NaOH]. If three times water is added to
3) 2.5x10-2 mole lit-1 sec-1 the reaction mixture, the rate of the
4) 2.5x10-8 mole lit-1 sec-1 reaction compared to the original rate will
52. For 2 NH 3 
Au
 N 2  3H 2 rate w.r.t N 2 is be

2  10  3 M min  1 , then rate w.r.t N 2 after 1


rd
1
th 3)
1
th 4) 16 times
1) 2)
20 min will be (in M min-1) 3 9 16
1) 2  103 2)  2  10 3 57. The concentration of reaction decreases
3) 104 4)  2  10 3 from 0.2M to 0.05M in 5 minutes. The rate
53. The specific rate of a reaction is 1.5×10-4 of reaction in mole.lit 1.sec 1 is
lit mole-1sec-1. If the reaction is commenced 1) 8.3x10-4 2) 0.05 3) 0.0005 4) 0.15
with 0.2 mole/lit of the reactant, the initial
58. A  B kA = 10 e
15 -2000/T
rate of the reaction in mole lit-1sec-1 is
C D kC = 1014 e-1000/T
1) 1.5  10 4 2) 3  1 0  5
Temperature T at which (kA=kC)
3) 6  1 0  6 4) 6  1 0  5
1) 1000K 2) 2000K
54. For the process 2A  products, rate of
reaction w.r.t A at 10th second is 2000 1000
3) K 4) K
2  1 0  2 M s  1 then rates of same process 2.303 2.303
at 5th and 15th seconds (order  0) 59. From the following data for the
respectively are (in M/s) decomposition of N2O5 at 300C, find out the
1) 101 & 4  102 rate constant(in min–1). Volume of O2 after
10 min. of the reaction=90ml. Volume of
2) 2.7 102 &1.6  102 O2 after completion of the reaction=100ml
3) 1.6  102 & 2.7  102 1) 2.303 2) 0.2303
4) 2 102 & 2 102 3) 0.02303 4) 23.03

CHEMICAL KINETICS 35
ACTIVE SITE EDUTECH - 9844532971
ACTIVE SITE EDUTECH - 9844532971

Sr |12th class CHEMISTRY VOL-2


60. If doubling the concentration of the 2) 1/4 th of the original rate
reactant A increases the rate by 4 times 3) 1/2 th of the original rate
and tripling the concentration of A 4) same as the original rate
increases the rate by 9 times, the rate is 65. For the decomposition reaction:
proportional to
N 2O4 g   2NO2 g  ; the initial pressure of
1) concentration of A
2) square of concentration of A N 2 O 4 falls from 0.46 atm to 0.28 atm in 30
3) under root of conc. of A minute. What is the rate of appearance of
4) cube of concentration of A
NO 2 ?
61. Consider a system containing NO2 and SO2
1) 12  10 2 atm.min 1
in which NO2 is consumed in the following
two parallel reactions 2) 1.2  10 2 atm.min 1
2NO2 
k1
 N2O4 ; NO2  SO2 
k2
 NO  SO3 3) 1.2  102 atm.min 1
The rate of disappearance of NO2 will be 4) 1.8  101 atm.min 1
equal to 66. The rate for decomposition of NH3 on
1) k1[ NO2 ]  k 2 [ NO2 ]
2
platinum surface is zero order. What are
2) k1[ NO2 ]2  k 2 [ NO2 ][ SO2 ] the rate of production of N 2 and H 2 in
3) 2 k1[ NO2 ]
2 mole.lit 1.sec 1
4) 2 k1[ NO2 ]2  k 2 [ NO2 ][ SO2 ] if k  2.5  10 4 mole.lit 1.sec 1
62. Consider the reaction, 2A  B  Products, 1) 3.75  104 ,1.25 104
When concentration of B alone was
2) 1.25 104 ,3.75 104
doubled, the rate did not change. When the
concentration of A alone was doubled, the 3) 1.25 104 ,3.75  104
rate increased by two times. The unit of
rate constant for this reaction is 4) 1.25  104 ,3.75  104
1) s 1 2) lit.mol1.s1 67. 1dm 3 of 2M CH3COOH is mixed with
3) Unitless 4) mol. lit 1.s1 1dm 3 of 3M ethanol to form ester. The
63. For a reaction, the rate constant is decrease in the initial rate if each solution
expressed as, k  A.e  40000/ T . The energy is diluted with an equal volume of water
of activation is would be
1) 40000 cal 2) 88000 cal 1) 2 times 2) 4 times
3) 80000 cal 4) 8000 cal 3) 0.25times 4) 0.5 times
64. The reaction 68. For a reaction, k  2  1013 e 30000/ RT . When
CH 3COOC2 H 5  NaOH 
log k  y  axis  is plotted against 1/T
CH 3COONa  C2 H 5OH
is allowed to take place with initial
 x  axis  , slope of line will be.......Cal
concentrations of 0.2 mole/lit of each 30000 30000
reactant. If the reaction mixture is diluted 1) 2)
4.6 46
with water so that the initial concentration
of each reactant becomes 0.1 mole/lit. The 30000 30000
3) 4)
rate of the reaction will be 2.303 4.6
1) 1/8 th of the original rate

36 CHEMICAL KINETICS
ACTIVE SITE EDUTECH - 9844532971
ACTIVE SITE EDUTECH - 9844532971

Sr |12th class CHEMISTRY VOL-2


69. The rate of a reaction doubles when its 74. Rate constant of a first order reaction is
temperature changes from 300K to 310K. 0.0693min–1. If we start with 20 mol L–1, it
Activation energy of such a reaction will is reduced to 2.5 mol L–1 in:
be (1) 10 min (2) 20 min (3) 30 min (4) 40 min
 R  8.314JK mol and log 2  0.3010
1 1
75. Select the rate law that corresponds to the
data shown for the following reaction:A +
1) 48.6kJ mol1 2) 58.5kJ mol1 BC
1 1
3) 60.5kJ mol 4) 53.6kJ mol
70. Give the following data for the reaction:
XYZ
X Y Rate  101 ms 1
1.0 M 1.0 M 0.25
2.0 M 1.0 M 0.50
1.0 M 2.0 M 0.25
(1) rate = k[B]3 (2) rate = k[B]4
1.0 M 3.0 M 0.25 3
Which one is the rate law equation? (3) rate = k[A][B] (4) rate = k[A]2[B]2
76. Time required to decompose half of the
1) Rate  k  X  Y  substance for nth order reaction is
inversely proportional to
2) Rate  k  X   Y 
0 1
(1) an+1 (2) an–1 (3) an–2 (4) an
3) Rate  k  X  Y 
0 77. Which of the following statements are
correct?
4) Rate  k  X  Y  (1) The order of a reaction is the sum of the
2

powers of all the concentration terms in


71. The activation energy for a reaction is
the rate equation.
9.0kcal/mol. The increase in the rate
(2) The order of a reaction with respect to one
constant when its temperature is increased
reactant is the ratio of the change of
from 298K to 308K is:
logarithm of the rate of the reaction to the
1) 10 % 2) 100 % 3) 50 % 4) 63 %
change in the logarithm of the concentration
72. At 300K rate constant for A  products of the particular reactant, keeping the
at t = 50 min is 0.02 s 1 , then rate constant concentration all other reactants constant
at t = 75 min and 310 K will be (in (3) Orders of reactions can be whole numbers
1 )
of fractional numbers.
s (4) The order of a reaction can only be
0.04 determined from the stoichiometric
1) 2) 0.04  25 equation for the reaction.
25
78. In the reaction A  B if the concentration
 0.02  of A is increased by four times, the rate of
3) 0.04 4)   the reaction becomes doubled, the order
 25 
of the reaction is
ORDER OF REACTION 1) Zero 2) 1 3) 1/2 4) 2
73. For the reaction: A  B  product . 79. In the reaction A+B  Products, if B is
dx dx taken in excess, then it is an example of
 k  A  B if
a b
 k , then the order 1) Second order reaction
dt dt
2) Zero order reaction
of the reaction is:
3) Fractional order reaction
(1) 4 (2) 3 (3) 1 (4) 0
4) First order reaction
CHEMICAL KINETICS 37
ACTIVE SITE EDUTECH - 9844532971
ACTIVE SITE EDUTECH - 9844532971

Sr |12th class CHEMISTRY VOL-2


80. The unit of rate constant for a second order 87. The units of rate of reaction and rate
reaction is constant are identical for a
1) lit.sec 2) lit.mol.sec 1) fraction-order reaction
2) zero-order reaction
3) mol1.lit.sec1 4) mol.sec
3) first-order reaction
81. A chemical reaction A+2B  AB2 follows 4) second-order reaction
in two steps 88. If a reaction obeys the following equation
A+B  AB(slow)
2.303 a
AB+B  AB2(fast) k log
Then the order of the reaction is
t  a  x  then the order is
1) 3 2) 2 3) 1 4) 0 1) 0 2) 1 3) 2 4) 3
82. The graph drawn between the reaction time 89. Units for the rate constant of first order
and which of the following concentration reaction is
term gives a straight line plot passing 1) sec-1 2) mol.lit-1
through origin for the first order reaction 3) lit.mol-1 4) mol3.lit-1.sec-1
90. A zero order reaction is one whose rate is
1 independent of
1) log x 2)
a  x  1) temperature of the reaction
2) the concentration of the reactants
a 1
3) log 4) 3) the concentration of the products
ax a  x 2 4) activation energy
83. The reaction that obeys the expression 91. The dimensions of rate constant of a
1 second order reaction involves.
t1 
2
ka the order of reaction 1) neither time nor concentration
2) only time
1) 0 2) 1 3) 2 4) 3
3) time and concentration
84. The rate equation for the hydrolysis of an
4) time square and concentration
ester in presence of NaOH is,
92. The decomposition of H2O2 is represented
rate=k[ester] [NaOH]. If the concentration as H2O2  H2O+O (slow)
of NaOH is increased by 100 times than (O)+(O)  O2 (fast).
that of ester, the order of the reaction will Then the order of the reaction is
be 1) 1 2) 2 3) 0 4) 3
1) 1 2) 2 3) 0 4) 3
d  NH3 
85. When molecules of type A react with 93. represents
molecules of type B in one-step process to dt
give AB2,the rate law is 1) Rate of formation of Ammonia
1) rate =k[A]1 [B]2 2) rate=k[A]2 [B]1 2) Rate of decomposition of Ammonia
3) rate=k[2A] [B] 4) rate=k[A] [B] 3) Rate of consumption of N 2
86. The rate expression for a chemical reaction 4) Rate of consumption of H 2
2NO 2 F  2NO 2+F 2 is given by rate
94. Which of the following is not a first order
=k[NO2F]
reaction
The rate determining step may be 1) Hydrolysis of an ester in acidic medium
1) 2NOF2  2NO2+F2 2) Decomposition of N2O5
2) NO2F +F  NO2+F2
3) Decomposition of H 2O2
3) NO2F  NO2+F 4) NO2+F  NO2F
4) Oxidation of nitric oxide

38 CHEMICAL KINETICS
ACTIVE SITE EDUTECH - 9844532971
ACTIVE SITE EDUTECH - 9844532971

Sr |12th class CHEMISTRY VOL-2


95. The order of a reaction 102. The rate expression for a reaction is
1) can never be zero 1 3
dx
2) can never be fraction  k [ A] 2 [ B ] 2 , the overall order of the
3) must be a whole number dt
4) can be an integer or a fraction or zero reaction is.
96. The order of a reaction can be predicted 1 3
with the help of 1) 2 2) 3) 4) 1
2 2
1) molecularity of the reaction 103. Which of the following statements is false?
2) activation energy of the reaction 1) A fast reaction has a large rate constant
3) rate equation of the reaction and short half-life
4) reaction rate 2) Half life depends on concentration of
97. For the reaction A  B the rate law is, reactants for first order reaction.
rate=k[A]. which of the following statement 3) For a first order reaction,the half-life is
is incorrect ? independent of concentration
1) The reaction follows first order kinetics 4) The half-life of a reaction is, the time
required for the reaction to go to half of the
2) The t 1 of reaction depends upon initial
intial concentration of reactants
2
104. The time for half change for a zero order
concentration of reactants
reaction is..................
3) k is constant for the reaction at a constant
1) proportional to the initial concentration
temperature 2) proportional to the square root of the initial
4) The rate law provides a simple way of concentration
predicting the concentration of reactants 3) independent of initial concentration
and products at any time after the start of 4) inversely proportional to the initial conc.
the reaction 105. Rate equation for a second order is
98. The decomposition of Cl2O is 2.303 a 1 a
1) k  log 2) k  log
1) explosive reactions t ax t ax
2) second order reactions 1 X 1 a
3) first order reactions 4) thermal reactions 3) k  t a(a  x) 4) k  t 2 (a  x)
99. CO=initial concentration of the reactant 106. The hydrolysis of ethyl acetate
Ct=concentration of the reactant at time t, 

k=rate constant of the reaction. Then the CH 3COOC2 H 5  H 2 O 


H
CH 3COOH C2 H 5OH
equation applicable for a first order is a reaction of
reaction is 1) Pseudo first order 2) Second order
1) Ct=C0e-kt 2) Ct=C0ekt 3) Third order 4) Zero order
107. Radioactive decay follows which order
C kinetics?
3) C0=Cte-kt 4) 0  1
Ct 1) zero 2) 1 3) 2 4) 3
100. In a first order reaction fraction of the total 108. A reaction involving two different reactants
concentration of the reactant varies with can never be
1) Second order reaction
time 't' is equal to
2) First order reaction
1 3) Unimolecular reaction
1) e kt 2) 100.434kt 3) n 4) e kt 4) Bimolecular reaction
2
101. For a first order reaction, if ‘a’ is the initial 109. What is the order of a reaction which has
concentration of reactant, then the half life a rate expression? rate  k  A  B 
3/2 1

time is
1) independent of a 2)  a 3 1
1) 2) 3) zero 4) None
3)  a2 4)  a3 2 2

CHEMICAL KINETICS 39
ACTIVE SITE EDUTECH - 9844532971
ACTIVE SITE EDUTECH - 9844532971

Sr |12th class CHEMISTRY VOL-2


110. The molecularity of a reaction will be 117. Which one of the following statement for
1) fractional 2) zero order of reaction is not correct?
3) positive whole number 4) negative 1) Order can be determined experimentally
111. Which of the following is wrong 2) Order of reaction is equal to sum of the
1) Order of the reaction is negative, positive
powers of concentration terms in differential
or fractional
2) Molecularity of the reactions is always rate law
equal to the sum of stoichiometric co- 3) It is not affected with stoichiometric
efficients coefficient of the reactants
3) The order of a reactions may be zero 4) Order can not be fractional
4) Half life is independent of the concentration 118. Which of the following relation is correct
of reactants in first order reaction for a first order reaction?
112. Which statement is correct ? (k=rate constant; r=rate of reaction;
1) Molecularity of a reaction is same as the c = conc. of reactant)
order of reaction 1) k  r  c 2 2) k  r  c
2) In some cases order of reaction may be
c r
same as the molecularity of the reaction 3) k  4) k 
3) Molecularity may be zero r c
4) Molecularity may be fractional 119. The rate constant of a reation is
113. Which of the following cannot be 175 lit 2 mol 2 sec 1 . What is the order of
determined experimentally. reaction
1) Order 2) Rate 1) first 2) second 3) third 4) zero
3) Rate constant 4) Molecularity 120. If the rate of reaction A  B triples on
114. Which of the following statements increasing the concentration of A by 9
regarding molecularity of the reaction is times, then the order of reaction is
correct? 1) 2 2) 1 3) 1/2 4) 4
121.The half life period of catalytic
1) Molecularity relates to mechanism of
decomposition of AB3 at 50 mm is found to
reaction be 4 hrs and at 100 mm it is 2.0 hrs. The
2) It cannot be negative or fractional order of reaction is
3) Molecularity of a complex reaction has two 1) 3 2) 1 3) 2 4)0
(or) more steps and each individual step 122. A substance initial concentration (a) reacts
has its own molecularity. according to zero order kinetics. What will
4) All are correct be the time for the reaction to go to
115. For a reaction 2A+3B  Products, the rate completion
law expression is given by a k a 2k
1) 2) 3) 4)
rate = k (A)1  B  . The order of the k a 2k a
2

123. The conversion of A to B follows second


reaction with respect to A,B and over all order kinetics. Doubling the concentration
order of reaction are of A will increase the rate of formation of
1)1,2,1 2)3,2,1 B by a factor
3)1,2,3 4)2,1,3 1) 4 2) 2 3) 1/4 4) 1/2
116. The rate of a certain reaction at different 124. The rate constant is numerically same for
times is as follows three reactions of first, second and third
Time 0 10 20 30 order respectively. Which one is true for
Rate 3.2 10 3.18 10
2 2 2 3.19  102 rate of three reactions, if concentration of
3.22 10
reactant is greater than 1 M:
The order of the reaction is 1) r1  r2  r3 2) r1  r2  r3
1) 1 2) zero 3) 2 4) 3 3) r1  r2  r3 4) All of these
40 CHEMICAL KINETICS
ACTIVE SITE EDUTECH - 9844532971
ACTIVE SITE EDUTECH - 9844532971

Sr |12th class CHEMISTRY VOL-2


125. For the chemical reaction A  products, Note:1) Both (A) and (R) are true (R) is
it is found that the rate increases by a the correct explanation to (A)
factor of 6.25, when the concentration of A 2) Both (A) and (R) are true but (R) is not
is increased by a factor of 2.5. The order the correct explanation to (A)
of this reaction with respect to A is: 3) (A) is true but (R) is false
1) 2.5 2) 2 3) 1 4) 0.5 4) Both (A) and (R) are false
126. The initial rates for gaseous reaction 131. Assertion (A) : Molecularity of a reaction
cannot be more than three
A  3B  AB3 are given below
Reason (R) : Probability of simultaneous
collision between more than three particles is
very less
132. Assertion (A) : The molecularity of a reaction
is a whole number other than zero, but generally
less than 3
Reason (R) : The order of a reaction is always
whole number
133. The molecularity of an elementary
reaction X  2Y  Products is
1) 1 2) 2 3) 3 4) 0
order of the reaction is 134. Which order reaction obeys the expression
1) zero 2) three 3) one 4) two
1
127. 3/4 th of first order reaction was completed t1/ 2  in chemical kinetics
k .a
in 32min,15/16 the part will be completed
in 1
1) 0 2) 1 3) 2 4) 1
1) 24 min 2) 64 min 3) 16 min 4) 32 min 2
128. Initial concentration of the reactant is 135. Half life of a zero order reaction is 250sec.
1.0M.The concentration becomes 0.9M, t 7 5 % , t1 0 0% of the reaction respectively in
0.8M and 0.7M in 2 hours,4hours and sec. are
6hours respectively ,then the order of
1) 500, 375 2) 375, 500 3) 300, 575 4) 575, 300
reaction is
1) 2 2) 1 3) zero 4) 3 136. The half life period of a first order chemical
129. Molecularity of the following elementary reaction is 6.93 minutes. The time required
for the completion of 99% of the chemical
reaction is 2NO  O 2  2NO 2
reaction will be (log2=0.3010)
1) 0.5 2) 1 3) 2 4) 3 1) 23.03 minutes 2) 46.06 minutes
130. For which of the following reactions the 3) 460.6 minutes 4) 230.3 minutes
molecularity and order of the reaction are 137. 75% of a first order reaction is completed
respectively two and two in 32 minutes. 50% of the reaction would
1) Ester hydrolysis in acid medium have been completed in
1) 24 mins 2) 16 mins
2) Inversion of cane sugar in acid aqueous
solution 3) 18 mins 4) 23 mins
138. The half life periods of four reactions
3) Hydrolysis of ethyl acetate in caustic soda labelled by A,B,C & D are 30sec,4.8
aqueous solution. min,180sec and 16 min, respectively. The
4) Decomposition hydrogen peroxide in acid fastest reaction is
solution 1) A 2) B 3) C 4) D

CHEMICAL KINETICS 41
ACTIVE SITE EDUTECH - 9844532971
ACTIVE SITE EDUTECH - 9844532971

Sr |12th class CHEMISTRY VOL-2


139. Half-life periods for a reaction at initial 145. The initial concentration of cane sugar in
concentrations of 0.1M and0.01 are 5 and presence of acid was reduced from 0.20 to
50 minutes respectively. Then the order of 0.10M in 5 hours and to 0.05M in 10 hours,
reaction is
1) zero 2) 1 3) 2 4) 3 value of k?  in hr 1 
140. 75% of a first order reaction was completed 1) 0.693 2) 1.386 3) 0.1386 4) 3.465
in 32 min. When was 50% of the reaction
completed? 146. The decomposition of CH3CHO occurs as
(1) 24 min (2) 16 min (3) 8 min (4) 4 min CH 3 CHO(g)  CH 4 (g)+CO(g), the
141. The rate expression for the reaction kinetic data provided is
A( g )  B( g )  C( g ) is rate  kC A2 C B1/ 2 . [CH3CHO] Rate(mol.lit-1.sec-1)
What changes in the initial 1.75 x 10-3 2.06 x 10-11
concentrations of A and B will cause the 3.5 x 10-3 8.25 x 10-11
rate of reaction to increase by a factor of
7.0 x 10-3 3.30 x 10-10
eight?
The rate expression thus can be given as
1) C A  2; CB  2 2) C A  2; CB  4
1) k[CH3.CHO] 2) k[CH3.CHO]2
3) C A 1; CB  4 4) C A  4; CB 1
3) k[CH3.CHO]3 4) k[CH3.CHO]1/2
142. For the reaction system:
147. Obseve the following data regarding
2 NO( g )  O2( g )  2 NO2( g ) volume is
2 NH 3  W
 N 2  3H 2
suddenly reduced to half of its value by
increasing the pressure on it. If the reaction Pressure(in atm) : 5 10 20
is of first order with respect to O2 and Half life (min) : 3.6 1.8 0.9
second order with respect to NO, the rate The units of k is
of reaction will
1) diminish to one - eight of its initial value 1) min 1 2) atm . m in  1
2) increase to eight times of its initial value 3) ( atm . min)  1 4) atm  2 . min  1
3) increase to four times of its initial value
4) diminish to one fourth of its initial value 148. For a first order reation t 0.75 is 1386
143. The rate constant of a first order reaction seconds, then the specific rate constant in
3 1
at 27 C is 10 min .The ‘temperature
0
sec 1 is.
coefficient’ of this reaction is 2.What is the
1) 103 2) 102 3) 109 4) 105
rate constant  in min 1  at 17 0 C for this
reaction? 149. For N 2O5  2 NO2  1/ 2O2 , it is found that
1) 103 2) 5  104 3) 2  103 4) 102 d
[ N 2O5 ]  k1[ N 2O5 ],
144. A reaction was found to be second order dt
with respect to the concentration of carbon
monoxide. If the concentration of carbon d d
[ NO2 ]  k2 [ N 2O5 ] ; [O2 ]  k3 [ N 2O5 ]
monoxide is doubled, with everything else dt dt
kept as same, the rate of reaction will: then
1) Remain unchanged 2) Tripled 1) k1  2k2  3k3 2) 2k1  4k2  k3
3) Increases by a factor four
4) Doubled 3) 2k1  k2  4k3 4) k1  k2  k3

42 CHEMICAL KINETICS
ACTIVE SITE EDUTECH - 9844532971
ACTIVE SITE EDUTECH - 9844532971

Sr |12th class CHEMISTRY VOL-2


150. For a given reaction of first order, it takes 2) 2.5103 mol. lit 1. s1
20 min for the concentrations to drop from
1.0 M to 0.6M. The time required for the 3) 5.0 102 mol.lit 1.s1
concentration to drop from 0.6 M to 0.36
M will be 4) 0.5102 mol. lit 1.s1
1) more than 20 min 2) less than 20 min
3) equal to 20 min 4) infinity 156. SO2 Cl2  SO2  Cl2 is a first order gas
151. The half life of a reaction is 46 minutes when
reaction with k  2.2  10 5 sec 1 at 3200 C .
the initial concentration of the reactant is
0.4 mole/lit and 92 minutes when the initial The percentage of SO2Cl2 decomposed on
concentration is 0.2 mole/lit. The order of heating for 90 minutes is:
the reaction is
1) 1.118 2) 0.1118
1) zero 2) 0.5 3) 2 4) 1
152. The rate constant of a reaction at 3) 18.11 4) 11.18
temperature 200K is 10 times less than the
157. For the reaction a A  x P when [A] = 2.2
rate constant at 400K. What is the
mM the rate was found to be 2.4 mM s 1 .
activation energy  E a  of the reaction?
On reducing concentration of A to half, the
(R= Gas constant)
rate changes to 0.6 m M s 1 . The order of
1) 1842.4R 2) 921.2 R
reaction with respect to A is
3) 460.6 R 4) 230. 3 R
153. The decomposition of ozone proceeds as 1) 1.5 2) 2.0 3) 2.5 4) 3.0
 O2  O (fast)
O3  158. The rates of a reaction at different times
are given below
O  O3  2O2 (slow)
the rate expression should be Time (in min) Rate

1) Rate  k O3 
2
2) Rate  k O3  O2 
2 1 0 2.8 x 10-2
10 2.8 x 10-2
3) Rate  k O3 O2 
20 2.8 x 10-2
4) Rate  k O3 O2 
1

30 2.79 x 10-2
154. 50% of a reaction is completed in 16min,
under similar conditions 75% of the The order of the reaction is
reaction is completed in 48min. Order of 1) 2nd order 2) zero order
the reaction will be
1) 3 2) 1 3) 2 4) 0 3) 3rd order 4) 1st order
155. For a process, A  B  products, the rate 159. The isomerization of cyclopropane to form
of the reaction is second order with respect propene(  CH 3  CH  CH 2 ) is a first
to A and zero order with respect to B. When order reaction. At 760K, 85% of a sample
1.0 mole each of A and B are taken in a of cyclopropane changes to propene in 79
one litre vessel, the initial rate is min. Calculate the value of the rate
1102 mol. lit 1. s1 . The rate of the constant.
reaction, when 50% of the reactants have
1) 2.42 min 1 2) 3.66 102 min1
been converted to products would be
1) 1102 mol.lit 1 . s1 3) 2.40 102 min1 4) 1.04 102 min1

CHEMICAL KINETICS 43
ACTIVE SITE EDUTECH - 9844532971
ACTIVE SITE EDUTECH - 9844532971

Sr |12th class CHEMISTRY VOL-2


160. For a reaction following first-order kinetics, 165. The reaction 2A  B is first order in A
which of the following statements are with a rate constant of 2.8  10 2 s 1. How
correct.
long will it take(nearly) for A to decrease
1) The time taken for the completion of 75% from 0.88 M to 0.22M ?
of the reaction is twice t1/2.
1) 60 s 2) 76 s 3) 50 s 4) 44 s
2) A plot of the reciprocal of the concentration 166. For a first order reaction with half-life of
of the reactants against time gives a straight 150 seconds, the time taken for the
line concentration of the reactant to fall from
3) The degree of dissociation is equal to 1ekt M / 10 to M / 100 will be approximately
1) 1500 sec 2) 900 sec 3) 500 sec 4) 600 sec
4) A plot of  A 0 /  A versus time gives a
167. In a first order reaction, 50 minutes time
straight line. is taken for the completion of 93.75% of a
161. t 1 / 4 can be taken as the time taken for reaction. Half life of the reaction is
the concentration of a reactant to drop to 1) 25 min 2) 12.5 min 3) 20 min 4)10 min
3/4 of its initial value. If the rate constant 168. In 69.3 min, a first order reaction is 50%
complete. How much reactants are left
for a first order reaction is k, t1/4 can be
after 161 min?
written as 1) 80% 2) 40% 3) 20% 4) 60%
1) 0.75/k 2) 0.69/k 3) 0.29/k 4) 0.10/k
169. Type - I Type - II
162. The rate constant is numerically the same
for three reactions of first, second and third 1
I) first order reaction a )  n1
order respectively. Which of the following a
is correct: II) zero order reaction b) Radio active
decay
1) if  A   1 then r1  r2  r3
III) trimolecular reaction c) Photochemical
2) if  A   1 then r1  r2  r3 reactions
IV) half life period of d) 2CO  O 2  2CO 2
3) if  A   1 then r3  r2  r1 4) All of these ‘n’ th order
163. 99% of a 1st order reaction completed in 1) I - a, II - b, III - c, IV - d
2.303 minutes. What is the rate constant 2) I - b, II - c, III - d, IV - a
and half-life of the reaction 3) I - c, II - d, III - b, IV - a
1) 2.303 and 0.3010 2) 2 and 0.3465 4) I - d, II - c, III - b, IV - a
3) 2 and 0.693 4) 0.3010 and 0.693 COLLISION THEORY
164. For a first order reation, (A)  products, 170. To increase the rate of a chemical reaction,
the concentration of A changes from 0.1 catalyst
M to 0.025 M is 40 min. The rate of 1) increase the activation energy
reaction when the concentration of A is 0.01 2) decrease activation energy
M is 3) reacts with products
1) 3.47  104 M min 1 4) do not changes the activation energy
171. The energy of activation of a reaction is
2) 3.47  10 5 M min 1
dependent on
3) 1.73  10 M min
 4  1
1) temperature 2) pressure
4) 1.73  10 5 M min 1 3) concentration
4) nature of reactants

44 CHEMICAL KINETICS
ACTIVE SITE EDUTECH - 9844532971
ACTIVE SITE EDUTECH - 9844532971

Sr |12th class CHEMISTRY VOL-2

172. If the activation energy of both the forward 180. For a given reaction which one is higher
and the backward reactions are equal than the rest among the following
, H of the reaction is 1) Average energy 2) Threshold energy
1) zero 2) +Ve 3) -Ve 3) activation energy 4) Normal energy
4)cannot be predicted 181. The value of energy of activation for radio
173. For the exothermic reaction A+B  C+D. active decay is
H is the heat of reaction and Ea is the 1) high 2) low 3)zero 4) moderate
activation energy. The activation energy 182. In arrhenius equation, the fraction of
for the formation of A+B will be effective collisions is given by
1) Ea 2) H 3) Ea+ H 4) H - Ea 1) K=Ae-Ea/RT 2) A 3) e-Ea/RT 4) RT
174. The rate constant (k1) of one reaction is 183. On increasing the temperature by 100C,
found to be double that of the rate constant 1) number of collisions get doubled
of (k 2 ) another reaction. Then the 2) value of rate constant does not change
relationship between the corresponding 3) energy of activation increases
activation energies of two reactions (E1 and 4) specific rate of the reaction gets doubled
E2) can be represented. 184. The threshold energy of a chemical reaction
1) E1 > E2 2) E1 < E2 depends upon
3) E1 = E2 4) E1 = 4E2 1) nature of reacting species
175. Collision theory is applicable to 2) temperature
1) Unimolecular reactions 3) concentration of species
2) Bimolecular reactions 4) number of collisions
3) Trimolecular reactions 185. Activation energy is _____ to rate of
4) Tetra molecular reactions reaction
176. The rate constant is given by the equation 1) directly proportional
k   .Ze  Ea / RT
Which factor should 2)inversely proportional
register a decrease for the reaction to 3) equal
proceed more rapidly? 4) not related
1) T 2) Z 3) Ea 4) P 186. The rate of a reaction can be increased in
177. The excess of energy required for the general by all the factors except
reactant molecules to undergo a reaction 1) using a catalyst
is 2) increasing the temperature
1) Potential energy 2) Kinetic energy 3) increasing the activation energy
3) Thermal energy 4) Activation energy 4) increasing the concentration of reactants
178. Threshold energy (TE), internal energy of 187. The energy of activation of positive
reactants (IE) and energy of activation catalyzed reaction as compared to that of
(AE) are related as an uncatalyzed reaction is
1) AE = TE + IE 2) TE = AE + IE 1) more 2) less
3) IE = AE - TE 4) TE = AE = IE 3) same 4) may be more or less
179. The energy to be possessed by the 188. For producing the effective collisions, the
molecule participating in the reaction to colliding molecules must posses
give the products 1) a certain minimum amount of energy
1) < activation energy 2) threshold energy 2) energy equal to greater than threshold energy
3) < average energy 3) proper geometry
4) threshold energy + average energy 4) threshold energy and proper orientation

CHEMICAL KINETICS 45
ACTIVE SITE EDUTECH - 9844532971
ACTIVE SITE EDUTECH - 9844532971

Sr |12th class CHEMISTRY VOL-2


189. In the equilibrium A  B   C  D , the 1) Reaction A  B is faster and more
activation energy for forward reaction is exothermic than reaction C  D
25k.cal/mole and that of backward reaction 2) Reaction C  D is faster than reaction
is 15k.cal/mole. Which one of the following A  B but less exothermic
statement is correct
3) Reaction C  D is faster and more
1) It is an exothermic process
exothermic than the reaction A  B.
2) It is an endothermic process
3) It is reaction for which H  0 4) Reaction C  D 2 1 2 times faster than
4) It is a sublimation process reaction A  B at the same temperature
190. For an exothermic chemical process,
192. For a reversible reaction A 
 B , which
occuring in two steps as
one of the following statements is wrong
i) A  B  X  slow  ii) X  AB  fast  from the given energy profile diagram
The progress of the reaction can be best
described by

1)

1) Activation energy of forward reaction is


greater than backward reaction
2) The forward reaction is endothermic
2) 3) The threshold energy is less than that of
activation energy
4) The energy of activation of forward
reaction is equal to the sum of heat of
reaction and the energy of activation of
backward reaction.
3) 4) All correct 193. For A+B  C+D H =-20 kJ mole-1. The
activation energy for the forward reaction
is 85 kJ. Then the activation energy for
191. The energy profile diagrams of two the backward reaction is ___
reactions are shown in the figure. Then 1) 65 kJ 2) 105 kJ 3) 85 kJ 4) 40 kJ
194. An endothermic reaction A  B has an
activation energy 15 K Cal/mole and
enthalpy change ( H ) of the reaction is 5
KCal/mole. The activation energy of the
reaction B  A is (in K cal/mole)
1) 20 2) 15 3) 10 4) 5

46 CHEMICAL KINETICS
ACTIVE SITE EDUTECH - 9844532971
ACTIVE SITE EDUTECH - 9844532971

Sr |12th class CHEMISTRY VOL-2

1) 2 2) 3 3) 2 4) 2 5) 2 10. With increase in the concentration of the


6) 2 7) 2 8) 1 9) 3 10) 2 reactants, the rate of the chemical rections
11) 3 12) 2 13) 3 14) 4 15) 3 increases due to more number of effective
16) 1 17) 3 18) 1 19) 3 20) 4 collision.
21) 1 22) 2 23) 4 24) 1 25) 2 11. Increasing the temperature of the substance
26) 1 27) 4 28) 3 29) 3 30) 3 increases the fraction of molecues, which collide
with energies greater than activation energy
31) 1 32) 3 33) 3 34) 4 35) 2
36) 2 37) 4 38) 2 39) 2 40) 3  Ea  Hence increases the rate of reaction.
41) 4 42) 2 43) 4 44) 3 45) 1 12. Due to lesser number of bond rearrangements
46) 3 47) 4 48) 3 49) 2 50) 3
51) 2 52) 1 53) 3 54) 2 55) 4
56) 3 57) 3 58) 4 59) 2 60) 2
61) 4 62) 1 63) 3 64) 2 65) 3
66) 2 67) 3 68) 4 69) 4 70) 3 13.
71) 4 72) 3 73) 4 74) 3 75) 1
76) 2 77) 1 78) 3 79) 4 80) 3
81) 2 82) 3 83) 3 84) 1 85) 1
14. Due to more number of bond rearrangements
86) 3 87) 2 88) 2 89) 1 90) 2 15. For the gaseous reactants units of rate are
91) 3 92) 1 93) 2 94) 4 95) 4
Atm.sec 1
96) 3 97) 2 98)2 99)1 100) 4 16. Slowest step is the rate determining step for
101)1 102)1 103) 2 104) 1 105) 3
which ' k 1 ' is least.
106) 1 107)2 108)3 109) 2 110) 3
18. Higher the rate constant. more will be the extent
111) 2 112) 2 113) 4 114)4 115) 3 of completion
116) 2 117) 4 118)4 119) 3 120) 3 19. Ionic reactions are faster than covalent reactions
121) 3 122) 1 123) 1 124)3 125) 2 21. Rate of a reaction increases with increase in
126) 4 127) 2 128)3 129) 4 130) 3 temperature
131) 1 132) 3 133) 3 134) 3 135) 2 22. As the reaction proceeds, the rate of the
136) 2 137) 2 138) 1 139) 3 140) 2 reaction decreases.
23. Molecular mass does not influence the rate of
141) 2 142) 2 143) 2 144) 3 145) 3
the reaction where as others influence
146) 2 147) 2 148) 1 149) 3 150) 3 24. Rate of the reaction is directly proportional to
151) 3 152) 2 153) 2 154) 3 155) 2 the product of active masses of the reactants
156) 4 157) 2 158) 2 159) 3 160) 1 28. mol1 n .lit n 1.sec 1
161) 3 162) 4 163) 2 164) 1 165) 3 30. Rate constant does not depend an activation
166) 3 167) 2 168) 3 169) 2 170) 2 energy
171) 4 172) 1 173) 3 174) 2 175) 2 31. The concentration of the reactants decreases
176) 3 177) 4 178) 2 179) 2 180) 2 while that of products increases with time.
181) 3 182) 3 183) 4 184) 1 185) 2 1 d  NO 2  1 d  NO d O2 
33. rate    
186) 3 187) 2 188) 4 189) 2 190)2 2 dt 2 dt dt
191) 3 192) 3 193) 2 194) 3 34. with decrease in the activation energy, the rate
of the reaction increases
CHEMICAL KINETICS 47
ACTIVE SITE EDUTECH - 9844532971
ACTIVE SITE EDUTECH - 9844532971

Sr |12th class CHEMISTRY VOL-2


35. Another form of Arrhenious equation 54. Rate of reaction decreases with time
Ea  1  55. Rate of reaction is same for all the reactents
36. ln k      ln A ; y  mx  c and products
R T 
56. r  k CH 3COOC2 H 5  NaOH  on adding,
37. r  k  A B 
three times of water the resultant volume
 A  B   16k becomes four times that of initial volume.
r1  k  A B 
1/ 4 1/ 4 change in conc.
57. Rate=
38. Rate of the reaction decreases with time time interval
 from A  products 
 1014 e1000/T 10  e
15 2000/T 1000/T
39. Rate does not depends on excess quantity of 58. 10 . e
reactant
40. The rate constant becomes double for every 2.303 a
0 0 59. k log
10 C rise in temperature. For 20 C rise, the t ax
rate constant will be 4 times
60. 2n  4, 3n  9
 k 1  4 k 2 or k 2  0.25 K 1
61. rate  k1  NO 2      1
2

d H2  d  I 2  1 d  HI 
41.    rate  k 2  NO 2 SO 2       2 
dt dt 2 dt
d  A 1 d  B 1 d  NO 2 
 k1  NO 2  ;
2

42.   2 dt
dt 3 dt
d  NO 2 
43. Increase in rate of reaction is maximum for the  k 2  NO 2 SO 2 
reaction having the maximum activation energy dt

 Ea 62. r  k  A 1  B0 First order : unit of k = sec1


44. Slope  63. k  Ae  Ea / RT
2.303R
45. Rate of reaction w.r.t NH3 =Rate of reaction 64. r  k CH 3COOC2 H 5  NaOH 
w.r.t O2 1 d  NO 2   0.28  0.46 
65. 
46. According to stoichiometric relation 2 dt 30
47. 40  2
 50 2
 60 2
 70  2
 80 1/ 2d  NO 2 
 6  103
rate : 5  2 = 80
4
dt
d  N 2  1 d H2  d  NO 2 
48.   1.2  102 atm.min 1
dt 3 dt dt
49. Unit of k  mole lit sec
1 n n 1  1
1 d  NH 3  d  N 2  1 d  H 2 
66.   
50. For every 100 C rising in temperature, rate of 2 dt dt 3 dt
reaction becomes double due to double the Rate  k  NH 3 
0

no. of effective collisions


d  NH 3 
51. r  k  A
2
 2.5 104 mol.ltr 1S1
dt
52. Given reaction is Zero order reaction.
For zero order rate remains constant with time. d  N2  1
  2.5 104  1.25 104
53. From units of rate constant, it is second dt 2
order Rate  k  reactant 
2
d H2  3
  2.5  104  3.75 104
dt 2
48 CHEMICAL KINETICS
ACTIVE SITE EDUTECH - 9844532971
ACTIVE SITE EDUTECH - 9844532971

Sr |12th class CHEMISTRY VOL-2


67.
85. A  2 B  AB2
CH3COOH  C2 H5OH  CH3COOC2 H 5  H 2O
86. The No. of species involved in rate
 2 moles   3moles  determination step is equal to stoichiometric
rate  k  CH3COOH  C2 H5OH  coefficient of concentration terms in rate
68. y  mx  c Slope equation.

30000 30000 87. Unit of rate constant for zero order reaction
 m   and unit of rate of the reaction are same (i.e)
2.303 R 2.303 2
mol.lit 1.sec 1
k 2 E a  T2  T1 
69. 2.303log    88. Rate expression for first order is
k1 R  T1T2 
2.303 a
k log
70. rate  k  X  Y 
0 t ax

k 2 E a  T2  T1  89. mol1 n .lit n 1.sec 1 for first order n = 1


71. 2.303log   
k1 R  T1T2  Hence the unit becomes sec 1
k2 9  10  90. For zero order reactions, the rate of the reaction
 2.303log   298  308  is independent of the concentration of the
k1 2  103
reactants.
k2 91. Expect for first order reactions, the unit of rate
  1.63 ; i.e. 63% increase
k1 constant depends as the dimension
72. For every 100 C temperature rate constant
almost doubled  mol 1 n
.lit n 1.sec 1  concentration and time
78. r  k A n  (1); 2 r  k 4 A n (2)
    92. Decomposition of H 2O2 is example of first
From eq (1) & (2) , we get n  1/ 2 order reaction.
79. For any reaction excess amount of reactant is 93. 2 NH  N  3H
3 2 2
taken then order w.r.t that reactant is zero.
80. mol1nlit n1 sec1 for n = 2, rate of decomposition of ammonia
1  3
d NH
it becomes mol 1lit 1 sec 1 . 
2 dt
81. Order of the reaction can be known from the
no. of reactants involved in rate determining step 94. 2 NO  O2  2 NO2 is an example of second
(slow step) order reaction whereas the remaining given are
first order reactions.
95. Conceptual.
96. Order of the reaction can be known by the rate
82. equation of the reaction
97. For first order reaction , t1/2 does not depend
1
83. t1/ 2  n 1 for n = 2, the expression becomes 0.693
ka on the initial concentration . t1/ 2 
k
1
t1/ 2 
ka 98. Decomposition of Cl2O is an example of
84. If the concentration of one of the reactant is second order reaction.
taken as excess then order w.r.t that reactant is
zero.
CHEMICAL KINETICS 49
ACTIVE SITE EDUTECH - 9844532971
ACTIVE SITE EDUTECH - 9844532971

Sr |12th class CHEMISTRY VOL-2


c0 122. For zero order reaction tcompletion  a / k
99. kt  2.303  log c
123. Rate  k  A 
t 2

c0 c
kt  ln ,   kt  ln t 124. r1  k  A  ;r2  k  A  ; r3  k  A 
1 2 3
ct c0
ct If  A   1 r3  r2  r1
 e  kt   kt
c0  ct  c0 e
125. Rate  k  A 
m

100.  ct  c0 e  kt , ct  e  kt
6.25 r  k  2.5A 
m

101. For first order reaction , t1/2 does not depend


126. According to given data  A  is constant w.r.t
0.693
on the initial concentration . t1/2 
k  B order is 2
1 3 3
102. order    2 th completion = 2t1/ 2
2 2 127.
4
103. For first order reactions, t1/2 is independent of
15
initial concentration th completion = 93.75%  4t1/2
16
1 a 128. In zero order the rate of reaction is independent
104. t1/ 2  n 1 for n = 0
 t1/ 2 
ka k of the concentration of the reactants.
106. Water is present in excess. Therefore, the rate 129. Given is elementary reaction, hence molecularity
of reaction is independent of conc. of H 2O is 3.

3 1 130. CH 3COOC2 H 5  NaOH 


109. Order =  1 
2 2 CH 3COONa  C2 H 5OH
110. Molecularity of the reaction is always positive
whole number but can not be zero, fractional r  k CH 3COOC2 H 5  NaOH 
or negative. Order = 2 molecularity = 2
113. Collision theory is applicable for bimolecular 131. Molecularity never exceeds ‘3’
reactions. 132. Molecularity is a whole number and never
115. Order w.r.to A = 1 Order w.r.to B = 2 exceeds 3’. Order may be zero or fractional
Over all order = 1 + 2 or whole number or negative.
116. For zero order, rate remains constant with time. 133. Three molecules are participating in elementary
117. Order can be fractional reaction.
1 1
118. r  k  conc 
n
t1/2  ,t 
n 1 1/2
134. a a
119. From units of rate constant, the reaction is third n 1  1  n  2
order
3
120. 3  9n 135.  i  t75%  t50%  ii  t100%  2t50%
2
n  1/ 2
10
121. t1/2  a1n 136. t 90%  t 50% t 99%  2t 90%
3
4   50 
1 n
137. t75%  2t50%
 2  2n1 n  2
2  100 
1 n

50 CHEMICAL KINETICS
ACTIVE SITE EDUTECH - 9844532971
ACTIVE SITE EDUTECH - 9844532971

Sr |12th class CHEMISTRY VOL-2


138. The reaction which is having low half life that is
k2 Ea  1 1 
fastest reaction. 152. log k  2.303R  T  T 
1  1 2 
n 1
'
t1/ 2  a2 
139. "    O  O3   2O2
slow

t1/ 2  a1 
153. r  k1 O O3   k1 kc O3 O3 
2.303 a O2 
140. k  log
32 a /4
 k O3  O2 
2 1
 O2  O
O3 
k
2.303
 2  0.301 kc 
O2 O
32
O3 
0.693
t1/2   16 min 154. For second order : t3/ 4  3t1/ 2
0.0433
155. k  1102 lit. mol 1.s1 rate = k[ A]2 [ B]0
141. Rate  k  A  B
2 1/2

When 50% of the reactants are converted into


142. Rate  k  NO   O 2  products
2

rate = 1102 (0.5)2  2.5 103 mol .lit 1.s 1


143. For every 100 C temperature rate constant
almost doubled 2.303 a
156. Use k  log to then with the help
t ax
144. Rate  k  CO 
2
of this calculate x
145. Given reaction is first order reaction n
r A
157. r  k  A  ......(1)
n
 k   ......(2)
0.693 4 2
 t1/ 2 
k  2
 4  2n n=2
146. From result (1) & (2) , as the concentration 1
increases by 2 times , rate becomes 4 times.
so it is second order. 158. For zero order reactions, the rate of the reaction
does not change with time.
147. Given reaction is zero order reaction
2.303 a
0.693 159. k  log
148. t75%  2t50% t50%  t ax
k
2.303 100
d d d O2   log
149. 2   N 2 O5    NO 2   4 79 100  85
dt dt dt
k  2.4  10 2 min 1
150. 1.0 M  0.6 M  40%  160. For first order t75%  2  t50%
0.6 M  0.36 M  40% 
2.303 a
161. k  log
For particular fraction of reaction, time taken t ax
is same in first order
2.303 a 2.303 4
'  log  log
t 1 n 1 t1/2 3a / 4 t1/ 4 3
a 
 2 
2
151. From given data t ''
 a1  2.303  0.125 0.29
1 k 
2 t1/4 t1/4
so the reaction is the second order

CHEMICAL KINETICS 51
ACTIVE SITE EDUTECH - 9844532971
ACTIVE SITE EDUTECH - 9844532971

Sr |12th class CHEMISTRY VOL-2


184. Threshold and activation energies depend on
162. r1  k  A  ;r2  k  A  ;r3  k  A 
1 2 3
the nature of the reactants
If  A   1 r3  r2  r1 185. Lesser the activation energy, more will be the
rate of the reaction.
If  A   1 r1  r2  r3
186. Activation energy is inversly proportional to
If  A   1 r1  r2  r3 rate of the reaction.
187. A positive catalyst increases the rate of the
2.303 a reaction by decreasing the activation energy
163. k  log
t ax 189. Activation energy forward reaction is greater
164. Rate = k  A  0.003477  0.01 than that of backward reaction .Hence the
reaction is endothermic
2.303 a 190. In graph 2, activation energy for X’ is small. so
165. k  log
t ax reaction from X  AB takes place fastly.
0.693 2.303 a 191. From C  D , activation energy is lesser than
166. t1/ 2  , k log
k t ax that of activation energy from A  B
167. 93.75% completion 4t1/ 2 are required 192. Threshold energy = activation energy + normal
Total time required is 50 min ; energy.
50 193. Activation energy for backward reaction
 t1/ 2 
4 = H  activation energy for forward
168. t1/2  69.3min for 2t1/2  138.6 min reaction.
For 161 min 20% is left 194. Activation energy for backward reaction
169. All radioactive disintegration reactions are first = H  activation energy for forward
1 reaction.
order and t1/ 2  n 1
a
170. A catalyst increases the rate of the reaction by
decreasing the activation energy
171. Activation energy depends on the nature of the
reactants
173.  Ea back wardreaction   Ea  forward reaction H
for exothermic reaction .
174. Higher the rate constant, lesser the activation
energy
176. Ea decreases
178. Threshold energy = Activation energy + Internal
energy
180. Threshold energy = Activation energy + Internal
energy
181. Because H  0
183. For every 100 C rise in temperature, the rate
of the reaction generally gets doubled

52 CHEMICAL KINETICS
ACTIVE SITE EDUTECH - 9844532971
ACTIVE SITE EDUTECH - 9844532971

Sr |12th class CHEMISTRY VOL-2

6. In the reaction 2NO  O 2  2NO 2 , if the


rate of disappearance of O2 is
16gm.min -1 , then the rate of appearance
of NO 2 is
RATE OF REACTION & FACTORS
1) 90 gm.min 1 2) 46gm.min 1
1. The increase in rate constant of a reaction
is more when the temperature increases 3) 28gm.min 1 4) 32gm.min 1
from 7. 2A  B  D  E For this reaction
1) 290 K - 300 K 2) 300 K - 310 K proposed mechanism
3) 310 K - 320 K 4) 320 K - 330 K A  B  C  D  slow  ,
d [ B]
2. For 3 A  xB, is found to be 2/3rd of A  C  E  fast  . The rate law expression
dt
for the reaction is
d [ A]
1) r  K  A  B 2) r  K  A B
2
. Then the value of ‘x’ is
dt
3) r  K  A 4) r  K  A C
2
1) 1.5 2) 3 3) 1/2 4) 2
3. For a chemical reaction Y2  2Z  8. For a reaction Ea  0 and k  3.2  10 4 s 1
Product, rate controlling step is at 300 K. The value of K at 310 K would
be
1
Y Z  Q. If then the concentration of 1) 6.4  10 4 s 1 2) 3.2  10 4 s 1
2
Z is doubled, the rate of reaction will 3) 3.2  108 s 1 4) 3.2  105 s 1
1) Remain the same 2) Become four times 9. The rate constant k, for the reaction
1
3) Become 1.414 times 4) Become double N 2 O5  g   2NO 2  g   O 2  g  is 2.3  10 2 s 1 .
2
4. Observe the following reaction
Which equation given below describes
2A  B  C The rate of formation of C is the change of  N 2 O5  with time?  N 2 O5 0
2.2  103 mol L1 min 1 . What is and  N 2 O5 t correspond to concentration
d A of N 2 O5 initially and at time t
the value of 
dt
 in mol L1
min 1  ?
1)  N 2 O5 t   N 2 O5 0  kt
1) 2.2  103 2) 1.1 10 3 2)  N 2 O5 0   N 2 O5 t ekt
3) 4.4  103 4) 5.5  103 3) log10  N 2 O5 t  log10  N 2 O5 0  kt
5. Assertion (A) : The rate of reaction can  N 2 O5 0
4) ln  kt
also increase w.r.t its product if one of the  N 2 O 5 t
products act as catalyst 10. The rate reaction for the reaction
Reason (R) : A catalyst lowers the 2A  B  C is found to be rate =k[A] [B].
activation energy of reactions. The correct statement in relation to this
1) Both (A) and (R) are true (R) is the correct reaction is that the
explanation to (A) 1) unit of k must be s 1
2) Both (A) and (R) are true but (R) is not 2) value of k is independent of the initial
the correct explanation to (A) concentration of A and B
3) rate of formation of C is twice the rate of
3) (A) is true but (R) is false
disappearance of A.
4) Both (A) and (R) are false 4) t1/ 2 is a constant
CHEMICAL KINETICS 53
ACTIVE SITE EDUTECH - 9844532971
ACTIVE SITE EDUTECH - 9844532971

Sr |12th class CHEMISTRY VOL-2


11. The rate law for a reaction between the (k) is 2 106 M 2 . S 1 . Then rate of the
substances A and B is given by
Rate= k  A n  Bm on doubling the reaction become 2 10 9 M 2 . S 1 only
concentration of A and halving the when
concentration of B, the ratio of the new rate A  B  C
to the earlier rate of the reaction will be as
1) 0.3 M 0.2 M 0.2 M
1 nm
1) 2) (m + n) 3) (n – m) 4) 2 2) 0.2 M 0.1 M 0.2 M
2 m n 
12. Hydrogenation of vegetable ghee at 250 C 3) 0.1 M 0.1 M 0.1 M
4) 0.2 M 0.2 M 0.1 M
reduces pressure of H 2 from 2
atmosphere to 1.2 atmosphere in 50 17. xA  yB  zC. If
minutes. The rate of reaction in terms of
d  A d  B d C 
molarity per second is    1.5 then x,y
dt dt dt
1) 1.09 10 6 2) 1.09 10 5
and z are:
3) 1.09 10 7 4) 1.09 10 8
1) 1,1,1 2) 3,2,3 3) 3,3,2 4) 2,2,3
13. For a reaction 2SO2  O2  2 SO3 rate of 18. For the complex
consumption of SO2 is 6.4  103 kg / sec. 
Ag   2 NH 3   Ag  NH 3 2 
the rate of formation of SO3 in same
units will be
 dx 
1) 6.4 103 2) 8 103
 dt 

   2  10 L mol s  Ag   NH 3 
7 2 2 1  2

3) 4 103 4) 16 103 
1 102 s 1  Ag  NH 3 2 
1
14. A gaseous reaction A2 ( g )  B ( g )  C ( g )
2
Hence, ratio of rate constants of the
shows increase of pressure from 100mm forward and backward reaction is:
to 120mm in 5 min. The rate of
disappearance of A2 is 1) 2 107 L2 mol 2 2) 2 109 L2 mol 2
1) 4mm min-1 2) 40mm min-1 3) 1 102 L2 mol 2 4) 0.5 109 L2 mol 2
-1 -1
3) 8mm min 4) 20mm min
19. The following reaction is first order in A
15. Concentration of a reactant ‘A’ is changed
and first order in B:
from 0.044M to 0.032M in 25 minutes,
the average rate of the reaction during this A  B  Product, Rate  k  A B 
interval is
1) 0.0048 mol.lit 1.min 1
2) 0.00048 mol.lit 1.sec 1
3) 4.8  104 mol.lit 1.min 1
4) 0.0048 mol.lit 1.sec 1
16. For 2A  B  C  products, rate law is
Relative rate of this reaction in vessel I
2
given by rate  k  A  B & rate constant and II of equal volume is:
1) 1:1 2) 1:2 3) 2:1 4) 1:4

54 CHEMICAL KINETICS
ACTIVE SITE EDUTECH - 9844532971
ACTIVE SITE EDUTECH - 9844532971

Sr |12th class CHEMISTRY VOL-2


25. For the reaction 2 A  3B  product, A is
20. For the reaction A Products,
in excess and on changing the
d  A concentration of B from 0.1 M to 0.4 M,
  k and at different time interval,
dt rate becomes doubled. Thus, rate law is:
[A] values are dx 2 3 dx
1)  k  A  B  2)  k  A B 
Time 0 5 min 10 min 15 min dt dt
[A] 20 mol 18 mol 16 mol 14 mol dx 0 2 dx 1
3)  k  A  B  4)  k  B2
dt dt
At 20 minute, rate will be:
1) 12 mol/min 2) 10 mol/min 26. A reaction is catalysed by H + ion; in
3) 8 mol/min 4) 0.4 mol/min presence of HA rate constant is
21. The decomposition of 2N2O5 2N2O4  O2 is 2  10 3 min 1 and in presence of HB rate
at 200C . If the initial pressure is 114 constant is 1 103 min 1 . HA and HB (both
mm and after 25 min. of the reaction the strong acids) have relative strength as:
total pressure of gaseous mixture is 133 1) 0.5 2) 0.002 3) 0.001 4) 2
mm. Calculate the average rate of the ORDER OF REACTION
reaction in 27. Which of the following represents the
1 1 1
 a  Atm.m  b mol.lit .s respectively expression for 3/4th life of first order
reaction
1) 0.002, 8.58 107 2) 0.001,8.58  10 7
2.303 2.303
3) 0.002,8.58  10 4 4) 0.001,8.58  10 3 1) log 3 / 4 2) log 3
k k
22. A  B & C  D are first order reactions,
2.303 k
ratio of t99.9% values is 4 : 1, then ratio of 3) log 4 4) log 4
k 2.303
rate constants K1 to K 2 is 28. For a reaction A+2B  products, when B
1) 4 : 1 2) 2 : 1 3) 1 : 1 4) 1 : 4 is taken in excess, then the rate law
23. A substance ‘A’ decomposes in solution expression can be written as
following first order kinetics. Flask I 1) Rate=k[A]1[B]0 2) Rate=k[A]1[B]2
contains 1L of a 1M solution of A and flask 3) Rate=k[A][B] 4) Rate=k[A]2[B]1
II contains 100 ml of a 0.6 M solution. After 29. The unit of rate constant obeying the rate
8 hours the concentration of A in flask I 1 2/3
expression r  k  A   B  is
has become 0.25. What will be the time
taken for concentration of A in flask II to 1) mole-2/3lit 2/3 time-1
become 0.3 M?
2) mole2/3 lit -2/3 time-1
1) 0.4 h 2) 2.4 h 3) 4.0 h
4) Can’t be calculated since rate constant is 3) mole-2/3lit -2/3 time-1
not given 4) mole 2/3lit 2/3 time -1
24. The energy of activation for a reaction is 30. For a reaction
3A   Pr oducts . It is
50kJ/mol. Presence of a catalyst lowers the
found that the rate of reaction becomes
energy of activation by 25%. What will be
nine times, if concentration of A is
the effect on rate of reaction at 300C . increased three times, then the order of
Other things remains same. the reaction is
1) 142.75 2) 242.75 3) 342.75 4) 442.75 1) 1 2) 2 3) 3 4) 1.414
CHEMICAL KINETICS 55
ACTIVE SITE EDUTECH - 9844532971
ACTIVE SITE EDUTECH - 9844532971

Sr |12th class CHEMISTRY VOL-2


31. The rate constant for a first order reaction 37. Type - I Type - II
is 60s 1. How much time will it take to I) SO2Cl2  SO2  Cl2 a) pseudo
reduce the initial concentration of the unimolecular
reactant to its 1 / 10 value?
th reaction
1) 3837 sec 1 2) 0.03837 sec 1 II) 2NO2  2 NO  O2 b) first order
reaction
3) 0.03837 min 1 4) 0.3837 sec 1 III) 2 NO  O2  2 NO2 c) second order
32. Units of rate constant of first and zero order reaction
reactions in terms of molarity (M) are 
IV) CH 3 COOC 2 H 5  H 2 O  H
respectively:
1) s 1 , Ms 1 2) s 1 , M CH 3COOH  C2 H 5OH d) third order
The correct matching is
3) Ms 1 ,s 1 4) M,s1 1) I-a, II-b, III-c, IV-d
33. For a given reaction, the half-life period 2) I-b, II-a, III-d, IV-c
was found to be directly proportional to the 3) I-d, II-c, III-b, IV-a
initial concentration of the reactant. The 4) I-b, II-c, III-d, IV-a
order of reaction is 38. For an elementary process,
(1) 0 (2) 1 (3) 2 (4) 3 1)The order and the molecularity are identical
34. A reaction which is of first order w.r.t 2)The order is greater than the molecularity
3)The order is lesser than the molecularity
reactant A , has a rate constant is 6 min 1 .If 4)The order is always fractional
we start with  A   0.5 mol.L , when 39. A  B  products is an elementary
1

reaction. When excess of A is taken in this


would  A  reach the value of 0.05 mol.L1. reaction,then the molecularity and order
1) 0.384 min 2) 15 min are respectively
3) 20 min 4) 3.84 min 1) 2 and 2 2) 2 and 1
3) 1 and 2 4) 1 and 1
35. For the reaction A  B  products, it is
40. A reaction involving two different reactants
found that order of A is 1 and order of B is (1) can never be a second order reaction
1/2.When concentrations of both A & B are (2) can never be a unimolecular reaction
increased four times the rate will increase (3) can never be a third order reaction
by a factor (4) can never be a first order reaction
1) 6 2) 8 3) 4 4) 16
41. The product of half life  t1/2  and the square
36. A  B  products, the kinetic data of the
of initial concentration of the reactant (a)

reaction is A mole lit
1
 
B mole lit1 is constant. Then the order of reaction is
1) 2 2) 3 3) 0 4) 1

Rate mole.lit Sec
1 1
 42. 50% completion of a first order reaction
takes place in 16 minutes, then the fraction
1) 0.5 0.5 2  10 4 that would react in 32 minutes from the
beginning
2) 0.5 1.0 1.99  10 4 1) 1/2 2) 1/4 3) 1/8 4) 3/4
3) 1.0 0.5 2.01 10 4 43. Out of 300g substance [decomposes as per
The order of the reaction is 1st order]. How much (nearly)will remain
1) one 2) zero after 18 hr?  t1/2  3hr 
3) two 4) fractional 1) 4.6 gm 2) 5.6 gm 3) 9.2 gm4) 6.4 gm
56 CHEMICAL KINETICS
ACTIVE SITE EDUTECH - 9844532971
ACTIVE SITE EDUTECH - 9844532971

Sr |12th class CHEMISTRY VOL-2


44. 75% of a first order process is completed 50. The reaction, 2A  B  Products, follows
in 30 min .The time required for 93.75%
completion of same process(in hr)? the mechanism 2 A  A2
1) 1 2) 120 3) 2 4) 0.25 A2  B  Products (Slow) The order of the
45. The half life of a radio active material is reaction is
one hour. What would be the time required
1
for 99.9% completion 1) 2 2) 1 3) 3 4) 1
2
1) 5 hours 2) 10hours
51. Diazonium salt decomposes as
3) 2 hours 4) 20 hours
46. Sucrose decomposes in acid solution into C6 H 5 N 2 Cl  C6 H 5 Cl + N 2
glucose and fructose according to the first At 0o C , the evolution of nitrogen becomes
order rate law, with t1/2  3.00 hours. two times faster when the initial
What fraction of sample of sucrose remains concentration of the salt is doubled
after 9 hours? therefore it is.
1) 1.250 M 2) 5.00 M 1) a first order reaction.
3) 0.125 M 4) 0.250 M 2) a second order reaction.
47. Rate expression for xA + y B  products 3) independent of the initial concentration of
is Rate  k[ A]m [ B]n . Units of k w.r.t A and the salt.
4) a zero order reaction
B respectively are s 1 and M 1.s 1 when 52. Using the data given below the order and
concentrations of A and B are increased rate constant for the reaction :
by 4 times, then
CH 3CHO  g   CH 4  g   CO  g  would be
1) R f  16 Ri 2) Ri  16 R f
3) R f  8 Ri 4) R f  64 Ri
48. A first order reaction was commenced with
0.2 M solution of the reactants. If the
molarity of the solution falls to 0.02M after
100 minutes the rate constant of the
reaction is
1) 2 x 10-2 min-1 2) 2.3 x 10-2 min-1
3) 4.6 x 10-2 min-1 4) 2.3 x 10-1 min-1
49. The experimental data for the reaction 1) 2,  k  2.0 l / mol sec 
2NO g   Cl2 g   2 NOCl g  are given 2) 0,  k  2.0 mol / l sec 
below
3) 2,  k  1.5 l / mol sec  4) 1,  k  1.5 sec 1 
53. For the first order reaction A  product.
When the concentration of A is
2.5  10 2 M the activation energy is
20K.Cal/mole. If the conc. of  A  is
doubled, at same temperature, the
activation energy becomes equal to
1) 40K.cal/mole 2) 10K.cal/mole
What is the order of the reaction ?
20
1) 1 2) 2 3) 3 4) 0 3) 20K.cal/mole 4) 2 RT K.cal/mole

CHEMICAL KINETICS 57
ACTIVE SITE EDUTECH - 9844532971
ACTIVE SITE EDUTECH - 9844532971

Sr |12th class CHEMISTRY VOL-2

54. A g   B g  is a first order reaction. The 58. For a reaction dC  k  H   n . If pH of


dt  
initial concentration of A is 0.2 mol.lit 1. reaction medium changes from two to one,
After 10 minutes the concentration of B is the rate becomes 100 times of the value at
1
found to be 0.18 mol.lit . The rate constant pH = 2. The order of reaction is
1) 1 2) 2 3) 0 4) 3
 in min 1  for the reaction is 59. For the reactions of I, II and III orders,
1) 0.2303 2) 2.303 3) 0.693 4) 0.01 k 1  k 2  k 3 when concentrations are
expressed in mol.litre-1 . What will be the
relation in k 1 , k 2 , k 3 ,if concentrations are
expressed in mol/mL ?
1) k1  k 2  k 3 2) k1  k 2  103  k 3  106
3) k1  2k 2  k 3 4) 2k1  3k 2  4k 3
55. 60. 900 ml of pure and dry O2 is subjected to
sielent electric discharge, so that after a
time 10 min. volume of ozonized oxygen is
found to be 870 ml. Now average rate of
Half-life is independent on concentration reaction in this interval is (in ml/min)
of A. After 10 minutes volume of N2 gas is 1) 3 2) 9 3) 90 4) 60
10 L and after completion of reaction 50 61. At some temperature, the rate constant for
L. Hence, rate constant is : the decomposition of HI on the surface of
2.303 1
gold is 0.08 MS1 2HI  g   H 2  g   I2  g 
1) log 5 min what is the order of the reaction. Howlong
10
will it take for the concentration of HI to
2.303 2.303
2) log1.25 min 1
3) log 2 min 1
drop from 1.50 M to 0.30 M.
10 10
1) zero order, t  7.5sec
2.303 2) zero order, t=15 sec
4) log 4 min 1
10 3) first order, t=22.5 sec
56. When the initial concentration is changed 4) first order, t=7.5 sec
from 0.50 to 1.0 mole lit , the time of half 62. For the first order gaseous reaction:
1

completion for a certain reaction is found x ( g )  2 y ( g )  z ( g ) the initial pressure,


to change from 50 sec. to 25 sec. Calculate
the order of reaction. Px  90 mm Hg . The pressure after 10
1) 1 2) 2 3) 3 4) 0 minutes is 180 mm Hg. The rate constant
of the reaction is:
57. For SO2Cl2( g )  SO2( g )  Cl2( g ) pressures
1) 2  10 3 sec 1 2) 2  103 sec 1
of SO2Cl2 at t = 0 and t = 20 minutes
3) 1.15  103 sec 1 4) 1.15  103 sec 1
respectively are 700 mm and 350 mm.
63. The half-life for the reaction.
When log ( P0 / P ) is plotted against time
1
(t), slope equals to N 2 O5  2NO 2  O 2 is 24hrs. at 300 C.
2
1) 1.505  10 2 s 1
Starting with 10g of N 2 O5 how many grams
2) 1.202  10 3 min 1
3) 1.505  10 2 min 1 of N 2 O5 will remain after a period of 96
hours?
4) 0.3465 min 1
1) 1.25 g 2) 0.625 g 3) 1.77 g 4) 0.5 g
58 CHEMICAL KINETICS
ACTIVE SITE EDUTECH - 9844532971
ACTIVE SITE EDUTECH - 9844532971

Sr |12th class CHEMISTRY VOL-2


64. The half life of a first order reaction 71. A  B  product.
A  B  C is 10 minutes. The from the following data calculate half life
concentration of ‘A’ would be reduced to period.
10% of the original concentration in
1) 10 minutes 2) 33 minutes
3) 90 minutes 4) 70 minutes
65. A first order reaction is half-completed in
45 minutes.How long does it need for
99.9% of the reaction to be completed?
1
1) 7 hours 2) 20 hours 1) 1.386 sec 1 2) 13.86 sec 1
2
3) 10 hours 4) 5 hours 3) 26.72 sec 1 4) 2.672 sec 1
66. For the zeroth order reaction, sets I and II
72. The gas phase decomposition of dimethyl
are given, hence x is :
ether follows first order kinetics:
CH3  O  CH3 ( g )  CH4 ( g )  H2 ( g)  CO( g)
The reaction is carried out in a constant
volume container at 5000 C and has a half
life is 14.5 minutes. Initially only dimethyl
ether is present at a pressure of 0.40 atm.
1) 2 min 2) 3 min 3) 4 min 4) 6 min What is the total pressure of the system
67. A  B , k1  0.693 sec 1 after 12 minutes? (Assume the ideal gas
behaviour.)
C  D , k 2  0.693 min 1 . If t1 & t2 are 1) 0.946 atm 2) 0.785 atm
half lives of two reactions, then 3) 0.777 atm 4) 0.749 atm
1) t1  t2 2) t1  60t2 73. The radioactive isotope 32 P decays by first
3) t2  60 t1 4) t2  2.303t1 order kinetics and has a half-life of 14.3
68. For 2 NH 3( g ) 
Pt
 products follows zero days. How long does it take for 95.0% of

a given sample of 32 P to decay?
order kinetics. If t1/ 2 at p = 4 atm is 25
1) 21 days 2) 42 days
sec, t1/ 2 at p = 16 atm will be (in sec) 3) 62 days 4) 80 days
1) 6.25 2) 625 3) 100 4) (25)1/ 4 74. 20% decomposition of H2O2 in presence
69. DDT on exposure to water decomposes of an acid requires 5 min. The time
according to first order kinetics. Half life required for 50% decomposition in minutes
= 10 years. How much time it will take for is
its decomposition to 99%? 1) 15.52 2) 1.552 3) 0.1552 4) 7.76
1) 50 years 2) 66.6 years 75. The first order rate constant for the
3) 500 years 4) 666 years
70. The rate of a first order reaction is 0.04 decomposition of N 2 O5 is 6.2 104 s 1 .
mol litre–1 sec–1 at 10 minute and 0.03 mol The half-life period for this decomposition
litre sec at 20 minute after initiation.
–1 –1
is :
Find the half life of the reaction.
1) 223.4 s 2)1177.7s
1) 2.406 min 2) 24.06 min
3) 240.6 min 4) 0.204 min 3)1117.7 s 4) 160.9 s

CHEMICAL KINETICS 59
ACTIVE SITE EDUTECH - 9844532971
ACTIVE SITE EDUTECH - 9844532971

Sr |12th class CHEMISTRY VOL-2


COLLISION THEORY 79. Effective collisions are those in which
76. Consider the energy profile, for the molecules must:
1) Have energy equal to or greater than the
reaction x  y  R  S .Which of the
threshold energy
following deduction about reaction is not 2) Have proper orientation
correct? 3) Acquire the energy of activation
4) All of these
80 The reaction A  C has activation energy
for the forward and the backward reaction
has 25kJ and 32kJ respectively. The  H
for the reaction is
1) 57 kJ 2) -57 kJ 3) 7 kJ 4) -7 kJ
81. Consider an endothermic reaction X  Y
with the activation energies E b and E f for
1) The energy of activation for the backward the backward and forward reactions
reaction is 80 KJ respectively. In general.
2) The forward reaction is Endothermic 1) E b  E f 2) E b  E f 3) E b  E f
4) There is no definite relation because
3) H for the forward reaction is 20 kJ
E b and E f
4) The energy of activation for F.R is 60 kJ 82. The rate constant of a reaction at 300 K is
77. The following figure denotes the energy 1.6x10-3 sec-1 and at 310K it is 3.2x10-3 sec- 1
diagram for a reaction the activation energy of the reaction
approximately in k.cals is
1) 12-13 2) 20-25 3) 30-40 4) 40-50
83. The rate constant, the activation energy
and the Arrhenius parameter of a chemical
reaction at 25C are 3  10 4 s 1 , 104.4 kJ / mol
and 6 1014 s1 respectively. The value of the
rate constant as T   is
1) 2.0 1018 s 1 2) 6.0 1014 s 1
3) Infinity 4) 3.6 1030 s1
Then the activation energy of the reverse 84. For an exothermic chemical process
reaction is occuring in two steps as
1) 2x 2) 2y 3) x  y 4) y  x  i  A  B  X  slow   ii  X  AB  fast 
78. The energies of activation for forward and The progress of the reaction can be best
described by
reverse reactions for A2  B2 
  2AB are
180 kJ/mol and 200 kJ/mol, respectively.
The presence of a catalyst lowers the
activation energy of both (forward and
reverse) reactions by 100 kJ/mol. The
enthalpy change of the reacton
A 2  B2  2AB in the presence of a
catalyst will be (in kJ/mol)
1) 300 2) 120 3) 280 4) -20

60 CHEMICAL KINETICS
ACTIVE SITE EDUTECH - 9844532971
ACTIVE SITE EDUTECH - 9844532971

Sr |12th class CHEMISTRY VOL-2


85. Given that the temperature coefficient for
1 d  A  d  C
the saponification of ethyl acetate by 4.  
NaOH is 1.75. Calculate the activation 2 dt dt
energy. 5. Catalyst lowers the activation energy of
reactions.
1) 1.0207 kcal 2) 10.207 kcal/mol.
3) 1.0207 cal 4) 10.207 cal/mol. d O2  1 d  NO 2 
 
86. What is the activation energy for the 6. dt 2 dt
decomposition of N 2O5 as, 7. Rate law depends upon slow step
8. If E a  0 then k = A = constant
1
N 2O5  2 NO2  O2 if the values of the 9. Decomposition of N 2 O5 is of first order
2
10. Given reaction is second order
rate constants are 3.45  105 and
11. r  k  A   B    (1)
n m

6.9  10 3 at 27 0 C and 67 0 C m
n 1 
respectively? r1  k  2A   B     (2)
1) 112.5 kJ 2) 200 kJ 2 
3) 149.5 kJ 4) 11.25 kJ  2  r1
  2n  m ; r1  2n  m.r
1 r
12. P  0.8atm / 50 min
n P
PV  nRT ; P  V RT ; M  RT
1)1 2) 4 3) 3 4) 3 5) 1 6) 2 13. According to Stoichiometric relation
7) 2 8) 2 9) 4 10) 2 11) 4 12) 2
13) 2 14) 3 15) 3 16) 3 17) 3 18) 2 x
100  x x
19) 2 20) 4 21) 1 22) 4 23) 3 24) 1 2
25) 4 26) 4 27) 3 28) 1 29) 1 30) 2 1 x
14. A2  B  C 100   120
2 2
31) 2 32) 1 33) 2 34) 1 35) 2 36) 2
37) 4 38) 1 39) 2 40) 2 41) 2 42) 4 x  40
43) 1 44) 1 45) 2 46) 3 47) 4 48) 2 dx 40
Rate =   8mm.min 1
49) 3 50) 3 51) 1 52) 1 53) 3 54) 1 dt 5
55) 2 56) 2 57) 3 58) 2 59) 2 60) 2
 0.044  0.032 
61) 2 62) 3 63) 2 64) 2 65) 1 66) 4 15. Average rate   
67) 3 68) 3 69) 2 70) 2 71) 1 72) 4  25
73) 3 74)1 75) 3 76) 1 77) 3 78) 4 16. According to given data  C is independent
79) 4 80) 4 81) 1 82) 1 83) 2 84) 3
1 d  A  1d  B 1 d  C 
85) 2 86) 1 17.  
3 dt 3dt 2 dt

kf 2  10 7
18. 
k b 1  10 2
19. When compared to vessel-1, In vessel-II the
concentration of A doubled. therefore rate
1. Rate constant is inversly proportional to becomes two times.
temperature. 20. For every 5 minutes change in concentration is
1 d A 1 d  B 2
2   2 moles per litre Rate   0.4
3 dt x dt 5

3. Rate  k  Y  Z 
1/ 2 21. 2Na 2xO5  2Nx2O 4  Ox /22

CHEMICAL KINETICS 61
ACTIVE SITE EDUTECH - 9844532971
ACTIVE SITE EDUTECH - 9844532971

Sr |12th class CHEMISTRY VOL-2

Pressure due to a moles = 114 mm 2.303 a


31. t log If
x k ax
Pressure due to a  moles = 133 mm
2 1
a  1 then  a  x  
x 10
Hence pressure due to moles = 133 - 114
2 dx
32. k   A  for first order  sec 1
and due to x moles 19  2  38 mm dt
 Pressure due to (a – x) moles = 114–76 = dx
38. k for zero order  M ol. lit  1 .sec  1
dt
38
Average rate =
25  760
 0.002 Atm m 1  A 0
33. For zero order reaction t1/2 
Again PV = nRT 2k
n 0.002 2.303 a
  34. k  log
V 0.0821 473  60 t ax
t1 k 2 35. Rate  k  A  B
22. t  k for first order reaction
2 1
1
23. The concentration of A remains 1/4th in 8 hours. R 1  k 1    1 ;
2
Therefore 1/  2 , n  2
n

1 
R 2  k  4  1   4   8
and E1/ 2 is 8  n  t1/ 2 ; t1/ 2  4; In 4 hours 0.6 2 
will become 0.3 36. For zero order reactions rate is independent of
24. k  Ae  Ea / RT concentrations.
3 3 37. See examples of different order reactions
k1  Ae 5010 / RT
k 2  Ae 37.510 / RT
38. For elementary process both order and
k1 molecularity same
 e  37.5 5010 / RT  e12.510 / RT
3 3

39. Molecularity is 2 order is 1


k2
40. As two different reactant molecules are reacting
k2 12.5 103 it can never be a unimolecular reaction.
2.303log 
k1 8.314  303 1
41. t 1  a n 1
2
k2 r2 k2
 142.75   142.75
k1 r1 k1 t1
42. is 16 Min.
2
25. Rate of reaction does not depends on excess
 32 Min is equal to two half lifes
quantity
43. In each half life time the amount becomes half .
2 hence18 hours is equal to 6 half lifes
26. Relative strength = ratio of rate constants =
1 44. 75% completion equal to 2 half lifes hence
2.303 a t1/2  15 min .
27. k  log
93.75 % completion equal to 4 half lifes
t ax
28. Rate does not depend upon the reactant which 45. t99.9%  10  t50%
is taken as excess quantity
46. 9 hours = 3t1/2
29. Units of k  mole1n .lit n 1.sec1
47. Acc to units w.r.t A is first order w.r.t B is second
30. rate  k  Re actant 
n order

62 CHEMICAL KINETICS
ACTIVE SITE EDUTECH - 9844532971
ACTIVE SITE EDUTECH - 9844532971

Sr |12th class CHEMISTRY VOL-2

2.303 a P0 Kt
48. k  log log  0
t ax P 2.303
49. W.r.t NO order is 2 and w.r.t Cl2 order is 1 0.03466
y  mx  c  m   0.01505
50. r  k [ A2 ][ B ] .............. (i) 2.303
n
[A ] 58. rb  k  H  
k  22
[ A] pH = 2  H    102
 [ A2 ]  k [ A] .................. (ii)
2
n
r0  k 10 2  at pH  1,  H    101
From (i) and (ii)
r  kk [ A]2 [ B ]  k '[ A]2 [ B ] r1 n n
r1  k 10 1  100  10  n  2
r0
51. r  k  A  ; 2 r  k  2 A 
n n
n 1
59. rate = k[Reactant]m
52. r  k CH 3CHO   (1) Let concentration of reactant be a mol.litre–1
n

m
then for I order : r1  k1  a  ..... (1)
4r  k  2CH 3CHO   (2)
n

If concentration of reactant be a mol mL–1, then


(2) 1 x
n2 k  concentration in mol litre1  a 103
(1) at (a  x)
1
53. It is first order reaction Thus, r1  k1 a 103  r1  k1 103 a 
2.303 a r1  k1'  a 
54. k  log ....(2)
t ax
Similarly for II order : r2  k 2  10 6  a 
2
55. T50 is independent of concentration of A. Hence
first-order reaction. 2
r2  k '2  a  ....(3)
A  N2  g 
3
At t = 0, a 0 Similarly for III order : r3  k 3 109 a 
At time t,(a-x) x = 10L 3
r3  k 3'  a  ....(4)
(after 10 minutes)
At complete Given, k 1  k 2  k 3
Reaction, (a-a) a = 50 L
k1' k '2 k 3'
  a  x   40 L    or
103 106 109
2.303 50 k1'  k '2  10 3  k 3'  10 6
k  log
10 40
60. 3O2  2O3 a  3x 2x
2.303
 log1.25 min 1 t  10 min 900  3x 2x
10 total volume after 10 min  900  x
n 1
T1  a 2  900  x  870 x  30 ml
56.   ;
T2  a1 
 volume O2 consumed in 10 min is
n 1
50  1.0 
  2   2
n 1
or 21   2 
n 1
3x means 90 ml
25  0.5 
dc 90
 1  n  1 . Hence n = 1 + 1 = 2. Reaction is of    9 ml / min
second order. dt 10
61. From the unit of k, the reaction is of zero order.
2.303 P 2.303 700
57. k  log 0 ; k  log  0.03466 C C
t P 20 350 k 0
t
CHEMICAL KINETICS 63
ACTIVE SITE EDUTECH - 9844532971
ACTIVE SITE EDUTECH - 9844532971

Sr |12th class CHEMISTRY VOL-2


1.5  0.30 A 
0.08  t  15S 2.303
t
Also, t  log  10 when t = 10 min
k A 
  20
2.303 a 2.303 90 2.303 4
62. k  log10 = log10 10  log
t a  x 10  60 90  45 k 3
(Note that pressure 180mm Hg corresponds 2.303 4
k  log  0.0288min 1
for x(g) unreacted 2y(g) formed and z(g) 10 3
formed) 0.693 0.693
t1/2    24.06min
 Pressure due to unreacted k 0.0288
180 m n
x  45 mm Hg 71.  Rate = k  A   B 
4
m n
 1.15103 sec1 . For case I : 0.05  k 0.1 0.2  .... (1)
63. 24 hours = 1t1/2 96 hours = 4t1/ 2 m n
For case II : 0.10  k  0.2   0.2  .... (2)
64. For 3t1/2 concentration decreases to 12.5% m n
For case III : 0.05  k 0.1  0.1 ... (3)
65. t 99.9%  10t 50%
m
66. By set I, half-life is 2 min. In set II. Number of 1 1
moles have been doubled thus half-life is also By Eqs. (1) and (2), 2   2   m=1
 
doubled, i.e, now it is 4 min. Thus
n
8 moles change to 4 moles in = 4 min and 4 1
moles change to 2 moles in = 2 min Thus, By Eqs. (1) and (3), 1   2   n=0
 
Total time = 6 min
1 0
0.693 Thus, Rate = k  A   B  O.R.  1  0  1
67. t 1 for A  B     
2
0.693 1 0
Now 0.05  k 0.1 0.2 
t 1 for 0.693
 CD
2 0.693  60 0.05
k   0.5sec 1
t ' 0.1
1
a' For I order reaction,
2

68. For zero order reaction t ''
a '' 0.693 0.693
1 t1/2    1.386sec1
2 k 0.5
0.693
69. k  10 yr 1 when t is 99% 72. CH 3OCH 3 g   CH 4  H 2  CO

2.303 a 2.303 t  0 0.4atm 0 0 0


= log = log102 t  12 0.4  x x x x
t a  0.99a t
total pressure 0.4  x  x  x  0.4  2 x atm
0.693 2.303
  log102 or
10 t 99% 2.303 a
k log
10 t ax
t  2.303  2  66.5
0.693 2.303 0.4
k log
years = 70 years 12 0.4  x
70. Rate = k   A  0.693
but k
0.04  k  A 10 and 0.03  k  A 20 14.5
A
 10 0.04 4 
0.693 2.303 0.4
   log
A 
  20
0.03 3 t1 12 0.4  x
2

64 CHEMICAL KINETICS
ACTIVE SITE EDUTECH - 9844532971
ACTIVE SITE EDUTECH - 9844532971

Sr |12th class CHEMISTRY VOL-2


x  0.175 atm
Ea 308  298
 2.303log10 1.75 
 total pressure= 0.4  2 x  0.4  2  0.175 1.987 308  298
 0.75 atm
2.303  308  298 1.987
73. Since it is first order reaction, therefore E a   log1.75calmol1
10
0.693 0.693
k = E a  10.207 kcal mol 1
t1/ 2 14.3
2.303 a 0.693 2.303 100 k E T  T 
k 86. log k  2.303R  T T 
2 a 2 1
log ;  log
t ax 14.3 t 5 1  1 2 
t  61.82days = 62 days
6.9 10 3 Ea  340  300 
0.693 log 
74. t1/ 2  3.45 10 5
2.303  8.31  340  300 
k
0.693 0.693 Ea  112.5 kJ
75. t1/2    1117.7 sec
k 6.2  10 4
76. For backward reaction
activation energy = 80 - 40 = 40kJ
77. Activation energy for backward reaction =x+ y
78. For exothermic reactions
1. A reaction proceeds by first order, 75% of
H  Ea of B.W.R  Ea of F.W.R this reactin was completed in 32 min. The
79. According to collision theory time required for 50% completion is
80. For exothermic reactions [BMU (Mains) 2007]
H  Ea of B.W.R  Ea of F.W.R (1) 8 min (2) 16 min
81. For endothermic reaction activation energy of (3) 20 min (4) 24 min
forward reaction is greater than activation 2. The temperature dependence of rate
energy of backward reaction. constant (k) of a chemical reaction is
written in terms of Arrhenius equation,
k E 1 1
82. log k  2.303R  T  T 
2 a
k  Ae Ea /RT . Activation energy (Ea) of the
1  1 2 
reaction can be calculated by ploting
83. k  Ae Ea /RT when T  , then k  A . [AIIMS 2007]
84. A dip in the curve shows the formation of X 1
1
(reaction taking place in two steps). Since the (1) log k vs (2) log k vs log T
T
reaction is exothermic Ea of X must be less.
k2 1
85. Given, k  1.75 (3) k vs T (4) k vs log T
1
3. Rate of a reaction be expressed by
T1  25C  25  273  298K,
following rate expression Rate = k[A]2[B],
T2  35C  35  273  308K if concentration of A is increased by 3 times
and concentration of B is increased by 2
(Since temperature coefficient is ratio of rate times, how many times rate of reaction
constants at 35C and 25C respectively.) increase [AMU 2007]
k 2 E a T2  T1  (1) 9 times (2) 27 times
 2.303log10 
k1 R T1T2 (3) 18 times (4) 8 times

CHEMICAL KINETICS 65
ACTIVE SITE EDUTECH - 9844532971
ACTIVE SITE EDUTECH - 9844532971

Sr |12th class CHEMISTRY VOL-2

4. If the volume of the vessel in which the oxygen will be obtained after a period
reaction 2NO  O2  2NO2 is occuring of 9.6 h ? [AIIMS 2007]
is diminished to 1/3rd of its initial volume. (1) 1.5 L (2) 3.36 L
The rate of the reaction will be increased (3) 1.05 L (4) 0.07 L
by [AMU (Med.) 2007] 9. Catalyst increses the rate of reaction
(1) 3 times (2) 9 times [AMU(Med.) 2007]
(1) by decreasing Ea (2) by increasing Ea
(3) 27 times (4) 36 times
5. The reaction of hydrogen and iodine (3) by decreasing A
monochloride is given as (4) by increasing entropy
H 2  g   2ICl  g   2HCl  g   I 2  g  10. Rate constant has the unit mol–2 L2 s–1,
This reaction is of first order with respect then order of reaction is [DUMET 2007]
to H2(g) and ICl(g), following mechanisms (1) zero (2) first (3) second (4) third
were propsed 11. The reaction follows the mechanism
Mechanism A :
 AB  fast 
k

A  B 1

H 2  g   2ICl  g   2HCl  g   I 2  g  k 1

Mechanism B  A  B2  slow  then rate law


k2
AB  B 
H 2  g   ICl  g   HCl  g   HI  g  ; slow is [DUMET 2007]
HI  g   ICl  g   HCl  g   I 2  g  ; fast (1) r  k  A B  (2) r  k  AB B 
Which of the above mechanism(s) can be
(3) r  k  A B  (4) r  k  A  B 
2 2
consistent with the given information about
the reaction [AIPMT 2007] 12. For the reaction
(1) B only (2) A and B both
(3) Neither A nor B (4) A only 2HX  H 2  X 2
6. In a first order reaction A  B , if k is d  HX  [DUMET 2007]
  rate
rate constant and initial concentration of dt
the reactant A is 0.5 M then the half-life is
[AIPMT 2007] 1 d  HX 
(1) rate w.r.t. HX  
2 dt
0.693 log 2
(1) (2)
0.5k k 1 d  HX 
(2) rate w.r.t HX  
log 2 In 2 2 dt
(3) (4)
k 0.5 k d  HX 
(3) rate w.r.t HX  
7. If 60% of a first order reaction was dt
completed in 60 min, 50% of the same d  HX 
reaction would be completed in (4) rate w.r.t HX  
dt
approximately (log 4 = 0.60, log 5 = 0.69) 13. Consider the following statements
[AIPMT 2007] The rate law for the acid catalysed
(1) 50 min (2) 45 min hydrolysis of an ester being given as
(3) 60 min (4) 40 min
Rate  k  H   ester   k ' ester . If the
8. The half-life for the reaction
acid concentration is doubled at constant
1
N 2O5  2NO2  O2 is 2.4 hours at STP. ester concentration
2 1. The second order rate constant, k is
Starting with 10.8 gr of N2O5, how much doubled

66 CHEMICAL KINETICS
ACTIVE SITE EDUTECH - 9844532971
ACTIVE SITE EDUTECH - 9844532971

Sr |12th class CHEMISTRY VOL-2


2. The pseudo first order rate constant, (1) – log10 [A] vs t (2) – loge [A] vs t
k is doubled (3) log10 [A] vs log t (4) [A] vs t
3. The rate of the reaction is doubled 19. The bromination of acetone that occurs in
Which of the above statements are correct acid solution is represented by this
[AMU (Med.) 2008] equation
(1) 1 and 2 (2) 2 and 3
CH3COCH3  aq   Br2  aq  
(3) 1 and 3 (4) 1, 2 and 3
14. A hypothetical reaction CH3COCH2Br  aq   H  aq   Br   aq 
A2  B 2  2AB follows the mechanism as These kinetic data were obtained for given
given below. reaction concentrations Initial
concentrations, M Initial rate,

A2   A  A  fast 
disappreance of Br2, Ms–1
A  B2  AB  B  slow 
A  B  AB  fast 
The order of the overall reaction is
[AMU(Med.) 2008]
1
(1) 2 (2) 1 (3) 1 (4) 0
2
15. Consider following two reactions
d  A Based on these data, the rate equation is
 k1  A
0
A  Pr oduct ,  
dt [AIPMT 2008]
d B  (1) Rate = k [CH3COCH3][H+]
B  Pr oduct ,   k2 B 
dt (2) Rate = k [CH3COCH3[Br2]
k 1 and k 2 are expressed in terms of (3) Rate = k [CH3COCH3][Br2][H+]2
molarity (mol L–1) and time (s–1) as
(4) Rate = k [CH3COCH3] [Br2][H+]
[BHU(Mains) 2008]
–1 –1 –1
(1) s , Ms L (2) Ms–1, Ms–1 20. The rate constants k1 and k2 for two
(3) s–1, M–1 s–1 (4) Ms–1, s–1 different reactions are 1016.e–2000/T and
16. What is the order of reaction which has a 10 15 .e –1000/T , respectively. The
temperature at which k1= k2 is
rate expression, rate  k  A 3/2 B 1 ?
[AIPMT 2008]
[AFMC 2007, 2008]
3 1 4 2000
(1) (2) (3) 0 (4) (1) 1000 K (2) K
2 2 2 2.303
17. T50 of first-order reaction is 10 min. 1000
Starting with 10 mol L–1, rate after 20 min (3) 2000 K (4) K
2.303
is [AIIMS 2008]
(1) 0.0693 mol L–1 min–1 21. The rate constant of a first order reaction
(2) 0.0693 × 2.5 mol L–1 min –1 at 27°C is 10–3 min–1. The tempeature
(3) 0.0693 × 5 mol L–2 min–1 coefficient of this reaction is 2. What is
(4) 0.0693 × 10 mol L–1 min–1 the rate constant (in min–1) at 17°C for
18. For a first order reaction, to obtain a this reaction ? [AFMC 2008]
–3
positive slope, we need to plot {where [A] (1) 10 (2) 5 × 10–4
is the concentration of rectant A} (3) 2 × 10–3 (4) 10–2
[AIIMS 2008]
CHEMICAL KINETICS 67
ACTIVE SITE EDUTECH - 9844532971
ACTIVE SITE EDUTECH - 9844532971

Sr |12th class CHEMISTRY VOL-2


22. The concentration of a reactant X
25. For the reaction, 2A  B  C  D the
decreases from 0.1 M to 0.005 M in 40
min. If the reaction follows first order order of reaction is [AMU 2009]
kinetics, the rate of the reaction when (1) one with respect to [B]
the concentration of X is 0.01 M will be (2) two with respect to [A]
[AFMC 2008] (3) three
4 –1 (4) cannot be predicted
(1) 1.73 × 10 M min
–4 –1
(2) 3.47 × 10 M min 26. In the reaction
(3) 3.47 × 10–5 M min–1
(4) 7.5 × 10–4 M min–1 BrO3  aq   5Br   aq   6H 

23. For a reversible reaction A 


 B, 3Br2 l   3H 2O l 
which one of the following statements is
The rate of apperance of bromine (Br2)
wrong from the given energy profile
is related to rate of disappearance of
diagram? [AMU 2008]
bromide ions as following [AIPMT 2009]

d Br2  5 d Br 

(1) 
dt 3 dt

d Br2  5 d Br 

(2) 
dt 3 dt
(1) Activation energy of forward reaction
d Br2  3 d Br 

is greater than backward reaction (3) 
(2) The forward reaction is endothermic dt 5 dt
(3) The threshold energy is less than that of
d Br2  3 d Br 

activation energy (4) 
(4) The energy of activation of forward reaction dt 5 dt
is equal to the sum of heat of reaction and 27. For the reaction A  B  products, it is
the energy of activation of backward observed that [AIPMT 2009]
reaction (1) on doubling the initial concentration
24. The plot between concentration versus time of A only, the rate of reaction is also
for a zero order reaction is represented doubled
by [AMU 2009] (2) on doubling the initial concentration
of both A and B, there is a change by
a factor of 8 in the rate of the reaction
The rate of this reaction is given by
(1) rate = k [A][B]2 (2) rate = k[A]2[B]2
(3) rate = k [A][B] (4) rate = k[A]2[B]
28. 3BrO  BrO3  3Br 

4 d BrO  2
If   k1 BrO  ;
dt

d BrO3  2
  k 2 BrO 
dt

68 CHEMICAL KINETICS
ACTIVE SITE EDUTECH - 9844532971
ACTIVE SITE EDUTECH - 9844532971

Sr |12th class CHEMISTRY VOL-2


33. For an endothermic reaction energy of
d Br   2
  k 3 BrO  ,
 activation is Ea and enthalpy of reaction is
dt –1
H (both in kJ mol ). Minimum value
the correct relation between k1, k2 and k3 of Ea will be: [CBSE (PMT) 2010]
is [AIIMS 2009] (1)  H (2)  H (3)  H (4) =0
(1) 3k1 = k2 = 2k3 (2) k1 = 3k2 = 1.5k3
34. The rate of reaction
(3) k1 = k2 = k3 (4) 2k1 = 3k2 = k3
2NO  Cl2  2NOCl is given by the rate,
29. For a reaction, the dimensions of rate
constant are same as that of rate, hence equation rate =k[NO]2[Cl2]. The value of
order of reaction is [AFMC 2009] the rate constant can be increased by:
(1) 0 (2) 1 (3) 2 (4) 3 [CBSE (PMT) 2010]
30. For a first order reaction, the time (1) increasing the temperature
required for 99.9% of the reaction to (2) increasing the concentration of NO
take place is nearly [DUMET 2009] (3) increasing the concentration of Cl2
(1) 10 times that required for half of the (4) doing all of these
reaction 35. For the reaction
(2) 100 times that required for two-thirds
1
of the reaction N2O5  g   2NO2  g   O2  g  , the rate
2
(3) 10 times that required for one-fourth
of the reaction of disappearance of N2O5 is 6.25×10–3 mol
L–1 s–1. The rate of formation of NO2 and
(4) 20 times that required for alf of the
reaction O2 will be respectively.
[CBSE(PMT) 2010]
31. Rate of the given reaction,
(1) 6.25×10 mol L s & 6.25×10–3mol
–3 –1 –1
(1) A  B  X
r1  0.05
L–1s–1
(2) X  B  r2  0.89
Y (2) 1.25×10–2 mol L–1 s–1 & 3.125×10–3mol
(3) Y  A 
r3  0.001 L–1s–1
 AY
(3) 6.25×10–3 mol L–1 s–1 & 3.125×10–3mol
(4) AY  B  r4  0.10
AYB will be L–1s–1
determined by [AMU(Med.) 2009] (4) 1.25×10–2 mol L–1 s–1 & 6.25×10–3 mol
(1) Step 1 : because the reaction starts with L–1s–1
the formation of X 36. Which one of the following statements
(2) Step 2 : because it is the fastest step for the order of a reaction is incorrect?
(3) Step 3 : because it is the slowest step [AIPMT(P) 2011]
(4) Step 4 : because it ends the reaction (1) Order of reaction is always whole
32. Which of the following is the correct number
statement [AMU(Med.) 2009]
(2) Order can be determined only
(1) Order of a reaction has always an
integral value experimentally
(2) Mechanism of a reaction proposed is (3) Order is not influenced by
always final stoichiometric coefficient of the
(3) Zero order reactions are multi-step reactants
reactions (4) Order of reaction is sum of power to
(4) Order of reaction can be predicted even the concentration terms of reactants
without knowing the rate law to express the rate of reaction
CHEMICAL KINETICS 69
ACTIVE SITE EDUTECH - 9844532971
ACTIVE SITE EDUTECH - 9844532971

Sr |12th class CHEMISTRY VOL-2

37. The activation energy of a reaction can


be determined from the slope of which to reduce to 5g? [NEET-2017]
of the following graphs?[AIPMT 2015] 1) 138.6 sec 2) 346.5 sec
3) 693.0 sec 4) 238.6 sec
1 T 1 43. The correct differences between first and
(1) ln k vs. (2) vs.
T ln k T second order reaction is that[NEET-2018]
1) The rate of first-order reaction does not
ln k depend on reactant conc. the rate of sec-
(3) ln k vs. T (4) vs.T
T ond order reaction does depend on re-
38. When initial concentration of a reactant actant conc.
is doubled in a reaction, its half-life 2) The half life of a first order reaction
period is not affected. The order of the does not depend on [A]0, the half life of
reaction is [AIPMT 2015] a second order reaction does dpend on
[A]0.
(1) Second 3) A first-order reaction can be catalysed;
(2) More than zero but less than first a second-order reaction cannot be
(3) Zero (4) First catalysed.
4) The rate of a first-order reaction does
39. The rate constant of the reaction A  B
depend on reactant concentration the
is 0.6  10 3 mole per second. If the rate of a second-order reaction does not
concentration of A is 5 M, then depend on reactant concentration.
concentration of B after 20 minutes is: 44. When initial concentration of the
[AIPMT 2015] reactant is doubled, the half-life period
of a zero order reaction [NEET-2018]
(1) 0.36 M (2) 0.72 M 1) is halved 2) is doubled
(3) 1.08 M (4) 3.60 M 3) is tripled 4) remain unchanged
40. The rate of a first-order reaction is 0.04 45. If the rate constant for a first order re-
action is K the time(t) required for the
mol  1s 1 at 10 seconds and 0.03 mol
completion of 99% of the reaction is
 1s 1 at 20 seconds after initiation of given by. [NEET-2019]
the reaction. The half-life period of the 1) t = 4.606/K 2) t = 2.303/K
reaction is: [NEET-1,2016] 3) t = 0.693/K 4) t = 6.909/K
(1) 54.1s (2) 24.1s (3) 34.1 (4) 44.1s 46. For the chemical reaction [NEET-2019]
41. Mechanism of hypothetical reaction. N 2 g   3H 2 g   2 NH 3 g  , the correct
x2  y2  2 xy, is given below:[NEET-2017] option is
i) x2  x  y  fast  d  N 2  1 d  NH 3 
1)  
dt 2 dt
ii) x  y2  xy  y  slow 
d H2  d  NH 3 
iii) x  y  xy  fast  2) 3 2
dt dt
The overall orderof the reaction will be
1) 2 2) 0 3) 1.5 4) 1 1 d H2  1 d  NH 3 
3)  
42. A first order reaction has a specific 3 dt 2 dt
reaction rate of 10–2 sec–1. How much
d  N2  d  NH 3 
time will it take for 20g of the reactant 4)  2
dt dt

70 CHEMICAL KINETICS
ACTIVE SITE EDUTECH - 9844532971
ACTIVE SITE EDUTECH - 9844532971

Sr |12th class CHEMISTRY VOL-2

1 d HX
12. 
2 dt
3 1
16. rate  k  A 3/2 B1   1 
1. (2) 2. (1) 3. (3) 4. (3) 5. (2) 2 2
6. (4) 7. (2) 8. (3) 9. (1) 10. (4)
kt
11. (3) 12. (2) 13. (2) 14. (3) 15. (4) 18. log Ct    log c o
2.303
16. (2) 17. (2) 18. (2) 19. (1) 20. (4)
19 Factual
21. (2) 22. (4) 23. (3) 24. (4) 25. (4)
26. (4) 27. (1) 28. (2) 29. (1) 30. (1) 20. k  A e  Ea /RT
31. (3) 32. (4) 33. (3) 34. (1) 35. (2) k 0 k 270 C
27 C
36. (1) 37. (1) 38. (4) 39. (2) 40. (2) 21. k 0  2 k170 C  2
41. (3 ) 42. (1) 43. (2) 44. (2) 45. (1) 17 C

46. (1) 2.303  x o


; Rate  k  x 
1
22. k  t
log
 x t
23. Threshold energy = Ea + Average energies of
reactant molecules TE  Ea
24. Ct = – kt + C0
t75% So plot of CtVs t will be a straight line with
1. t50%  negative slope
2
25. Order is experimental fact.
Ea
2. log k  log A 
1 d Br2  1 d  Br 

2.303 RT
26. 
3. Increase in rate of reaction = 32 × 21 = 18 3 dt 5 dt
times 27. (1) order w.r.t ‘A’ is 1
(2) overall order is 3  order w.r.t ‘B’ is 2
rate  k No  O2 
2
4.
1 d BrO  d BrO3  1 d Br 
  

 1 28.   
 32  2  27 time  c   3 dt dt 3 dt
 v
29. Dimension for rate constant
5. Factual
0.693 2.303 log 2 ln 2
‘k’  conc 1n time 1
6. t1/2    So rate constant for zero order and rate of
k k k
reaction will have same dimension
2.303 a
7. k log 30. t99.9%  10 t1/2
t ax
8. 108 gr of N2O5 = 11.2 lit O2 33. E a  H for endothermic
10.125 gr of N 2O5  1.05 lit O2 Ea greater or less than H for exothermic
9. Catalyst decreases Ea 34. k  Ae  Ea /RT

 conc.
1 n d N2O5  1 d NO2  2d O2 
10. Unit of k  35.   
time dt 2 dt dt
11. r = k [A][B] 36. Order of reaction may be fractional.
37. k  A.e  Ea / RT
k eq 
 AB Taking log on both side
k
 A B  1 Ea
k 1 ln k  ln A 
RT

CHEMICAL KINETICS 71
ACTIVE SITE EDUTECH - 9844532971
ACTIVE SITE EDUTECH - 9844532971

Sr |12th class CHEMISTRY VOL-2

Ea 1 3
 Order   1   1.5
m= 2 2
lnk R
42. For a first order reaction,
1
T

K
2.303
log
 A0
38. T1/2 for first order reaction is independent
from initial concentration of reactions t  At
ln2 2.303 20
T1/ 2  102  log
k t 5
39. Lets assume the reaction to be zero order
2.303  0.6020
 rate  k 102 
t
Moles of A reacted = moles of B formed
 [A] reacted t  138.6 sec
   3  60  20  0.72M 0.693
43. For 1st order reaction, t1/2  which
r2 C2 K
40. r  C  for first order reaction  is independent of initial concentration [A]0.
1 1
1
1 C 1 r For 2nd order reaction, t1/2 
k n 2  n 2 K  A0
t2  t1 C1 t2  t1 r1
Half-life depends on initial concentration
1 0.04 1 4 of reactant.
k n  n
20  10 0.03 10 3
 A0 
n 2 n 2 44. t1/2 
t1/ 2    10 2K
k n 4 / 3
As the half life on a zero order reaction is
2.3  0.3 directly proportional to initial concentration
10
2.3  0.6  0.477   It [A0] doubled then, t1/2 = doubled.
=24.4sec 0.693
45. t1/2 
41. x2  x  y  fast  K
10
x  y2  xy  y  slow  99% completed 2  t1/2
3
slow step is the rate determing step
10 0.693
Rate  K  x  y2   2 
3 K
Equilibrium constant for fast step,
4.606
t
 x
2
K
K
 x2  46. N 2  3H 2  2 NH 3

 x  K  x2  d  N 2  1  H 2  1 d  NH 3 
  
dt 3 dt 2 dt
By substituting  x  in equation (i), we get
d  N 2  1 d  NH 3 
Rate  K K  x2  y2   K ' x2   
 y2 
1/2
dt 2 dt

72 CHEMICAL KINETICS
ACTIVE SITE EDUTECH - 9844532971
ACTIVE SITE EDUTECH - 9844532971

Sr |12th class CHEMISTRY VOL-2


5. Consider a first order gas phase
decomposition reaction given below :
A g   B  g   C  g  The initial
1. The role of a catalyst is to change pressure of the system before
1)gibbs energy of reaction. decomposition of A was pi After lapse of
2) enthalpy of reaction. time ‘t’, total pressure of the system
3) activation energy of reaction. increased by x units and became ' pt ' The
4) equilibrium constant rate constant k for the reaction is given as
2. In the presence of a catalyst, the heat _________.
evolved or absorbed during the reaction
1) increases. 2) decreases. 2.303 pi
1) k  t
log
pi  x
3) remains unchanged.
4) may increase or decrease. 2.303 pi
3. Activation energy of a chemical reaction 2) k  t
log
2 pi  pt
can be determined by
1) determining the rate constant at standard 2.303 pi
temperature. 3) k  t
log
2 pi  pt
2) determining the rate constants at two
temperatures. 2.303 pi
3) determining probability of collision. 4) k  t
log
pi  x
4) using catalyst. 6. According to Arrhenius equation rate
4. Consider Figure and mark the correct
constant k is equal to Ae  Ea / RT . Which
option. of the following options represents the
1
graph of ln k vs ?
T

1) Activation energy of forward reaction is


E1 + E 2 and product is less stable than
reactant.
2) Activation energy of forward reaction is
E1 + E 2 and product is more stable than
reactant.
3) Activation energy of both forward and
backward reaction is E1 + E 2 and reactant
is more stable than product.
4) Activation energy of backward reaction
is E1 and product is more stable than
reactant.
CHEMICAL KINETICS 73
ACTIVE SITE EDUTECH - 9844532971
ACTIVE SITE EDUTECH - 9844532971

Sr |12th class CHEMISTRY VOL-2


9. Which of the following statements is not
correct about order of a reaction.
1) The order of a reaction can be a
fractional number.
2) Order of a reaction is experimentally
determined quantity.
7. Consider the Arrhenius equation given 3) The order of a reaction is always equal to
below and mark the correct option the sum of the stoichiometric coefficients
k= Ae a E / RT of reactants in the balanced chemical
equation for a reaction.
1) Rate constant increases exponentially
with increasing activation energy and 4) The order of a reaction is the sum of the
decreasing temperature. powers of molar concentration of the
2) Rate constant decreases exponentially with reactants in the rate law expression
increasing activation energy and decreasing
temperature. 10. Consider the graph given in Fig. Which of
3) Rate constant increases exponentially the following options does not show
with decreasing activation energy and instantaneous rate of reaction at 40th
decreasing temperature. second?
4) Rate constant increases exponentially with
decreasing activation energy and increasing V5 -V2 V4 -V2 V3 -V1 V3 -V1
1) 2) 3) 4)
temperature. 50-30 50-30 40-30 40-20
8. A graph of volume of hydrogen released 11. Which of the following statements is
vs time for the reaction between zinc and correct?
dil.HCl is given in Fig .On the basis of this 1) The rate of a reaction decreases with
mark the correct option
passage of time as the concentration of
reactants dereases
2) The rate of a reaction is same at any time
during the reaction.
3) The rate of a reaction is independent of
temperature change.
4) The rate of a reaction decreases with
increase in concentration of reactant(s)
12. Which of the following expressions is
correct for the rate of reaction given
below?
V3  V2 5Br -  aq  +BrO3-  aq  +6H +  aq 
1) Average rate upto 40s is
40
 3Br2  aq  +3H 2 O 1
V3  V2
2) Average rate upto 40 seconds is Δ  Br -  Δ  H +  Δ  Br -  6 Δ  H + 
40  30 1) =5 2)   =
Δt Δt Δt 5 Δt
V3
3) Average rate upto 40 seconds is Δ  Br -  5 Δ  H + 
40 3) =
Δt 6 Δt
V3  V1
4) Average rate upto 40 seconds is Δ  Br -  Δ  H + 
40  20
4) =6
Δt Δt

74 CHEMICAL KINETICS
ACTIVE SITE EDUTECH - 9844532971
ACTIVE SITE EDUTECH - 9844532971

Sr |12th class CHEMISTRY VOL-2


13. Which of the following graphs 4) Molecules should collide with sufficient
represents exothermic reaction? threshold energy and proper orientation
for the collision to be effective.
16. A first order reaction is 50% completed
in 1.26× 1014 s. How much time would it
take for 100% completion?
1) 1.26×1015s 2) 2.52×1014s

3) 2.52×10 28s 4) infinite


17. Compounds ‘A’ and ‘B’react according
to the following chemical equation.

A  g  +2B  g   2C  g 

Concentration of either ‘A’ or ‘B’ were


changed keeping the concentrations of
one of the reactants constant and rates
were measured as a function of initial
concentration. Following results were
obtained. Choose the correct option for
the rate equations for this reaction.
Experi Initial Initial Initial
ment concetration concentration rate of
of of formation of
1) a only 2) b only 3) c only 4) a and b [A]/mol L-1 [B]/mol L-1 [C] mol L-1 s -1
14. Rate law for the reaction A + 2B  C is 01. 0.30 0.30 0.10
found to be Rate=k[A][B] Concentration 02. 0.30 0.60 0.40
of reactant ‘B’ is doubled, keeping the 03. 0.60 0.30 0.20
concentration of ‘A’ constant, the value 1) Rate k=  A  [B] 2) Rate=k[A]  B
2 2

of rate constant will be


4) Rate=  A   B
2 0
1) the same 2) doubled 3) Rate =k [A][B]
3) quadrupled 4) halved 18. Which of the following statement is not
15. Which of the following statements is correct for the catalyst?
incorrect about the collison theory of 1) It catalyses the forward and backward
chemical reaction? reaction to the same extent.
1) It considers reacting molecules or atoms 2) It alters G of the reaction.
to be hard spheres and ignores their 3) It is a substance that does not change
structural features.
the equilibrium constant of a reaction.
2) Number of effective collisions 4) It provides an alternate mechanism by
determines the rate of reaction. reducing activation energy between
3) Collision of atoms or molecules reactants and products.
possessing sufficient threshold energy
results into the product formation.

CHEMICAL KINETICS 75
ACTIVE SITE EDUTECH - 9844532971
ACTIVE SITE EDUTECH - 9844532971

Sr |12th class CHEMISTRY VOL-2


19. The value of rate constant of a pseudo 21. The time for half life period of a certain
first order reaction ____________. reaction A  products is 1 hour. When
1) depends on the concentration of reactants
the initial concentration of the reactant
present in small amount.
2) depends on the concentration of reactants ‘A’ is 2.0 mol L1 , how much time does it
present in excess. take for its concentration to come from
3) is independent of the concentration of 0.50 to 0.25 mol L1 if it is a zero order
reactants.
reaction ?
4) depends only on temperature.
1) 4 h 2) 0.5 h 3) 0.25 h 4) 1h
20. Consider the reaction A  B. The
22. The rate of chemical reaction doubles
concentration of both the reactants and
the products varies exponentially with for every 100 C rise of temperature. If
time. Which of the following figures the temperature is raised by 500 C , the
correctly describes the change in
rate of the reaction increases by about
concentration of reactants and products
1) 24 times 2) 32 times 3) 64 times 4) 10
with time?
times
23. A reactant (A) forms two products
A 
k1
B , Activation energy Ea1 ,

A 
k2
 C , Activation energy Ea2 . If
Ea2  2 Ea1 , then K1 and K 2 are related as
1) k 1  2 k 2 e E a
2
/ RT
2) k 1  k 2 e E a
1
/ RT

dC
 k  A  B 
2
3) k 2  k 1 e E a
2
/RT
4)
dt
1
24. For a reaction A  2 B , rate of
2
disappearance of ‘A’ is related to the rate
of appearance of ‘B’ by the expression
d  A 1 d  B d  A 1 d  B 
1)   2)  
dt 2 dt dt 4 dt
d  A d  B d  A d  B
3)   4)  4
dt dt dt dt
25. For the reaction A  2 B  C  D  2 E the
rate equation is rate = k  A  B 0 C  then the
rate is
i) Doubled when [A] is doubled keeping
B and C constant
ii) Doubled when [C] is doubled keeping
A and B constant
iii) The same when [B] is doubled keeping
A and C constant

76 CHEMICAL KINETICS
ACTIVE SITE EDUTECH - 9844532971
ACTIVE SITE EDUTECH - 9844532971

Sr |12th class CHEMISTRY VOL-2


iv) Doubled when [B] is doubled keeping The percentage distribution of B and C are
A and C constant 1) 80% B and 20%C
The correct combination is 2) 76.83% B and 23.17% C
1) i, ii, iii 2) All are correct 3) 90% B and 10%C 4) 60% B and 40% C
3) ii, iv 4) ii, iii, iv 30. The rate of the reaction between
26. The acid hydrolysis of ester is: haemoglobin (Hb) and carbon
(i) first order reaction monoxide (CO) was studied at 200 C .
(ii) bimolecular reaction The following data were collected with
(iii) unimolecular reaction all concentration units in  mol / L (A
(iv) second order reaction haemoglobin concentration of 2.21
The true statements are  mol / L is equal to 2.21 106 mol/L )
1) i, ii 2) All are correct
3) ii, iv 4) ii, iii, iv
27. Which of the following statements are
correct:
(i) law of mass action and rate law
expressions are same for single step Determine the orders of this reaction
reaction with respect to Hb and CO and rate
(ii) the slowest step of a complex reaction constant.
gives the order of the complex reaction 1)1st order in Hb and Ist order in CO
(iii) both order and molecularity have 0.140 L mol 1 s 1
normally a maximum value of 3 2) 1st order Hb and 1st order in CO
(iv)molecularity of a complex reaction 0.280 L  mol 1 s 1
A+2B  C is 3 3) 1st order, 2nd order, 0.35 L mol 1s 1
1) i, ii, iii 2) All are correct
3) ii, iv 4) ii, iii, iv 4) 2nd, order, 2nd order, 0.24 L mol 1s 1
28. Consider the following reactions at 300 K 31. Consider following graphs (1) and (2)
X  Y  uncatalysed reaction 

X 
catalyst
 Y  catalysed reaction 
The energy of activation is lowered by
8.314 KJ.mol 1 for the catalysed reaction.
The rate of reaction is
1) 28 times 2) 15 times 3) 25 times
4) 22 times that of uncatalysed reaction. The order of reaction and the value of
29. A substance undergoes first order rate constant is:
decomposition. The decomposition follows 1) First order, 2.37  105 min 1
two parallel first order reaction as :
2) Second order, 2.37 105 torr 1 min 1
3) Zero order, torr 1 min 1
4) None of the above
1
32. Graph between log k and is a straight
T
  1
line with OX = 5, tan    2.303  . Hence
 
Ea will be
CHEMICAL KINETICS 77
ACTIVE SITE EDUTECH - 9844532971
ACTIVE SITE EDUTECH - 9844532971

Sr |12th class CHEMISTRY VOL-2


24. According to rate equation
25. r  k  A B  C 
0

26. In acid hydrolysis of ester. water is taken


in excess quantity r  k  Ester  order = 1
molecularity = 2
27. Molecularity is always a whole number
5 28. k uncat  A e  E uncat / RT
1) 2.303  2 cal 2) cal
2.303 k Cat  A e  E cat / RT
3) 2 cal 4) none of these
33. In a first order reaction the Let the E a for uncatalysed reaction be A,
concentration of product 'x' at time 't' then for catalysed reaction it will be
is given by the expression (a=initial  A  8.314 103  calaries/mol
concentration, k=rate constant,  A 8.314103  /RT
n=order) k cat e 3
  A/RT
 e8.31410 /RT  e3.3  28
1 k uncat e
1) x  a 1  e  kt  2) x 
a  x  29. For parallel path reaction
1 a k average  k1  k 2  1.26 10 4  3.8 10 5
3) x  n 1 4) x 
2 a  x   1.64 104 sec1
Also, Fractional yield of B
k B 1.26 104
= k   0.7683
av 1.64 104
Fractional yield of A
1. (3) 2. (3) 3. (2) 4. (1) 5. (2) kA 3.8 105
=   0.2317
6. (1) 7. (4) 8. (3) 9. (3) 10. (2) k av 1.64 104
11. (1) 12. (3) 13.(1) 14. (2) 15. (3) 30. Compare that 1 and 2 in the table. It is
16. (4) 17. (2) 18. (2) 19. (1) 20. (2) clear when the concentration of Hb is
double, the rate is also doubled, hence it is
21. (3) 22.(2) 23. (2) 24. (2) 25. (1) first order with respect to Hb, further an
26. (1) 27. (1) 28. (1) 29. (2) 30. (2) examination of data 2 and 3rd shows that
31 (2) 32. (3) 33. (1) it is also of first order with CO.
Therefore Rate of reaction
= [ Hb]1[CO ]1  Rate constant
0.619 mol L s 1
= 2.2 mol / L  1.00 mol / L  Rate constant
x 0.619
21. For a zero order reaction k  .....(1)  Rate constant =
2.2N
t
 A 0 = 0.280 L mol s 1 1

k 1 1
for zero order reaction 2t 1 .....(2)
31. p  kt  p
2 0

since  A0 = 2 M, t 1  1 h , k  1 Ea
32. log k = log A 
2 2.303RT
 from equation (1) Ea 1
0.25 Slope = 
2.303R 2.303
 given 
t  0.25 h
1 E a  2.303R  slope  R  2cal.
22. For every 100 C temperature raises rate of
33.  a  x   ae kt ; x  a 1  e  kt 
reaction becomes doubled
23. By based on arrhenius equation

78 CHEMICAL KINETICS
ACTIVE SITE EDUTECH - 9844532971

You might also like